Pediatric Case Studies and Diagnoses
Pediatric Case Studies and Diagnoses
Doc 1
1. A 1-month old baby boy was taken for immunisation. According to the mother, the baby
feeds well on the breast and is active. On examination, the infant was pink, his pulse rate
was 140 (is normal; range 120-170) beats per minute. On auscultation, there is a machinery
murmur, grade 3/6 in the left upper parasternal area. State the diagnosis of the baby.
2. A 5 months old boy was brought to Klinik kesihatan by his mother with pallor which she
noticed since he was two months old. The baby was exclusively breast fed since birth. On
examination the baby looked pale and had a grade 2/6 murmur in the left parasternal area.
Liver was palpable 3.5 cm below the right costal margin, and the spleen was just palpable.
Blood count results showed Hb 7g/dL, total white cell count 9.0 x 10^3/mL and platelet
counts 170. Blood film microcytic hypochromic anaemia. What is the best diagnosis test
A. Hb electrophoresis
B. Serum Ferritin
C. Serum iron
D. Total binding capacity
A. Scarlet Fever
B. Neonatal tetanus
C. Infectious mononucleosis
D. Viral C hepatitis
5. A 12 year old boy comes to the emergency department with a history of recurrent swelling
of the left knee joint for the last 2 years. Each episode of swelling improved after he was
given Factor VIII infusion. His maternal grandfather has the similar symptoms but the
swelling often increased in the right elbow. State the type of inheritance of the disease in the
patient.
A. Autosomal recessive
B. Autosomal dominant
C. X-linked recessive
D. DNA linked recessive
6. Baby was born with cataract, patent ductus arteriosus and blueberry muffin lesion on the
skin
A. Rubella
B. Toxoplasma
C. Cytomegalovirus
D. Herpes simplex
7. A 5 year old girl had high fever for 6 days, left cervical lymphadenopathy measuring 6 cm,
red and inflamed tongue and pharynx, non-purulent conjunctiva and swollen palms. What is
the diagnosis
A. Kawasaki Disease
B. Scarlet Fever
C. Measles
D. Acute viral conjunctivitis
Fits criteria for kawasaki: high grade fever >5 days, +⅘ of the following: lymphadenopathy,
strawberry tongue, conjunctivitis non purulent, rash, swelling desquamation redness of
extremities etc
8. a 6 week old boy came to the KK for vaccination. Early check up found out that the baby
had a large protruded umbilical, large anterior fontanelle, and jaundice for 6 weeks. What is
the investigation that needs to be done?
9. A 8 months old patient presented with rapid breathing, cough and poor oral intake with low
grade fever. Examination showed intercostal and subcostal recession and auscultation
heard prolonged expiration and generalised rhonchi. Blood pressure 110/80 Respiratory rate
55 temp 38.5°C. what are the most likely causative organism
A. Adenovirus
B. RSV
C. Filovirus
D. Pneumococcal
10.Child with diarrhoea for 2 days. Sunken fontanelle, thirsty and drink vigorously and
vomiting. Weight is 9.3kg (10kg 3 days before)
A. No dehydration
B. Mild Dehydration
C. Moderate dehydration
D. Severe dehydration
< 5% = mild
5-10%= moderate
> 10 % = severe
According to tanner staging, breast development starts first; stage 2: breast budding occur
12. 2 year old girl with intermittent fever for 3 days. urine dipstick shows negative leukocyte
esterase, positive nitrate. How to manage
13. A 5 year old child visits the neuro paediatric department. Have problem standing up.
Uses hands to push the floor before standing up. Which of the following is true.
For standing up etc, muscles closer to the trunk are used. Hip extensors / flexors (?)
14. Patient got positive HbS antigen, positive IgM antigen and negative anti-HbS
A. Acute hepatitis B
B. Chronic hepatitis B x
C. Hepatitis B with immunisation x
D. Resolved acute hepatitis B x
Doc 2
1. 6 year old aboriginal child, never been vaccinated before. Which vaccination should be
given first when she goes to the clinic for her first visit?
A.
B. Hep B, DTaP, MMR, varicella
C. Hep B, DT, MMR, IPV
D.
2. A 15 year old girl has a two week history of fatigue and back pain. She has widespread
bruising, pallor and tenderness over the vertebrae and both femurs. She has a liver palpable
4 cm below the costal margin and a hard spleen measuring 6cm. FBC shows Hb 50g/L,
leukocyte 2x10^8/L, and platelet count of 15 x 10^9/L. She should be investigated for
A. Infectious mononucleosis
B. Acute leukaemia
C. Meningococcemia
D. Idiopathic thrombocytopenic purpura
3. A 10 year old girl has been admitted with a history of low grade fever intermittently and
breathlessness for the past 3 weeks. She was operated for congenital heart disease at the
age of 18 months and again at 7 years. On examination she is pale and has a temperature
of 38°c. Her pulse rate is 128 beats per min, respiratory rate 22 breaths per min, and BP
104/60mmHg. On auscultation of the precordium, there is a grade 3 ejection systolic murmur
heard best in the left 4th intercostal area, and spleen is palpable 3 cm, rest of the systems
are normal. The most likely diagnosis is
A. Infective endocarditis
B. Cardiac failure
C. Pleural effusion
D. Pericarditis
Cannot definitively say IE, need to do further Ix (echo & blood c&s)
4. Sivan is a 7 year old boy who has had fever for 5 days and lethargy since this morning.
He had abdominal pain and vomiting yesterday but no diarrhoea. On examination, he is
conscious, lethargic and afebrile. His pulse rate is 138 beats per minute and his respiratory
rate is 34 per min. His peripheries are cold and blood pressure is 86/58mmHg. He has
guarding in the right hypochondrium but no other positive findings. An additional clinical
finding characteristic of dengue fever is
5. You are asked for advice for a baby boy who has undergone circumcision one day prior
and now he is oozing from the circumcision site. The baby was born via SVD at 38 weeks
gestation after an uncomplicated pregnancy and is doing well. His mother has no other
medical problem and is not on medication. When questioned, the mother reports that her
brother (infant’s maternal uncle) has a bleeding problem for which he is on medication
several times every week. There is no significant paternal history. Factor deficiency most
likely to affect this baby is
7. A 6 year old boy is brought to A&E by his parents in acute respiratory distress. He is
known to be asthmatic. His mother says that he has needed his inhaler a lot more than
normal over the past 24 hours. On examination, he is tachypneic with the respiratory rate of
50 breaths per minute. He is using his accessory muscle of respiration, and has an audible
wheeze. He is able to talk only in single words. His oxygen saturation is 85% and his heart
rate is 160 beats per minute. On arrival in A&E he was started on high flow oxygen and has
already been nebulized with salbutamol. The next step in management should be:
8. 5 month old Sarah is referred by her general practitioner with pyrexia. Her parents report
that she has been having a spiking temperature for the past 2 days and is feeding less than
usual. She has been vomiting although there is no history of diarrhoea. She has been
previously fit and well. On examination, she has a temperature of 38.8°c. She is sleeping but
easily arousable. She has a normal cry, normal movements and a soft fontanelle. She has a
full, wet nappy. Systemic examination does not reveal any abnormalities and her vitals are
normal. What further test will be most appropriate at this time?
A. CXR AP view
B. Full blood count
C. Nasopharyngeal aspirate for PCR
D. Urine for microscopy and culture
A. CT brain
B. Thyroid function test
C. ASO titre
D. Urinalysis
Dx: AGN; hx of skin infection (do urinalysis to look for RBC casts; which are pathognomonic
for AGN, to confirm dx)
10. A 16 month old child is brought from a village with a history of passing 10-15 loose stools
daily for the last 2 days. He also had 5-6 vomiting on the first days of illness. On examination
he is afebrile, drowsy, with cold extremities and has a capillary refill time of 6 seconds. His
mother says that she cannot remember when he last passed urine. His blood investigations
are likely to reveal
A. Hypokalemia
B. Hyperglycemia
C. Hypophosphatemia
D. Metabolic acidosis
- Patient is in hypovolemic shock -> poor perfusion -> increased anaerobic metabolism -> inc
lactate and metabolic acidosis
Doc 3
1. A 6 year old boy with ALL in the induction phase has fever, cough, and runny nose since
this morning. His brother has a fever and runny nose as well. On PE, body temp: 38oC.
Child is alert. There is no nasal flaring. On auscultation, there is prolonged expiration. Hb is
110g/L, WBC 2x103/uL, Platelet 150x103 /uL. The appropriate immediate mx is:
2. A baby boy was delivered by a 25 years old mother via elective LSCS. The baby looks
well and active with no resp distress. Lab ix of the mother shows HBsAg positive, anti-HBc
positive and anti-HBs negative. The appropriate feeding advice is:
A. Breastfeeding
B. Formula milk
C. Milk from a milk bank
D. Dependant on the baby’s ix results
A. CXR
B. Echocardiography
C. FBC
D. Viral serology
4. An 8 year old boy has right knee joint swelling that he noticed this morning. Two days
ago, he had a painful left wrist joint swelling that has now disappeared. 2 weeks ago, he had
a sore throat. On PE, the boy is alert, he has a swollen right knee joint that is red, and
painful on movement. A subcutaneous nodule is palpated along the midline at the back of
the patient. The diagnosis is: *****
5. A 6 years old boy comes with pallor, fever, and gum bleeding for 5 days. On PE, his temp:
38oC, liver and spleen are enlarged. Hb is 80 g/L; WBC 20x109/L; platelet count: 100 x 109/L.
The investigation that would confirm the diagnosis is:
A. Dengue serology
B. Blood film morphology- presence of blast cell
C. Bone marrow examination - confirmatory : bone marrow increase in cells by
lymphoblast
- 20% lymphoblast in the bone marrow
D. Factor VIII assay
ALL
PE - hepatosplenomegaly
6. A 5 months old infant has high fever, lethargy, seizures, and loss of consciousness.
Examination of CSF confirms the dx of meningitis. The earliest clinical sign of meningitis in
this infant would be:
8. A 2 year old child has had loose stools for the past 3 days. She is the 4th child of a rubber
tapper father and a home-maker mother. Her weight is 7kg (below 3rd percentile) and height
is 78cm (below 3rd percentile). She is afebrile, but lethargic. Her pulse is 126 bpm, and of low
volume. Her RR is 40/min, hands and feet are cold with a CRT of 4 to 5 seconds. She is
pale. The goals for her immediate mx should be to:
A. Administer 700ml of IV NS
B. Give ORS, keep her warm, and feed her
C. Administer iron, multivitamins, and zinc for her pallor
D. Administer bolus IV NS 20 ml/kg over 20 minutes.
Rapid pulse with low volume, cold peripheries, prolonged CRT, lethargic → patient in
compensated hypovolemic shock
Fluid resuscitation → IV 10 - 20 ml/kg of NS given bolus over 30 mins
9. A 4 month old infant presents with diarrhoea for 3 days and vomiting 5 to 6 times on the
1st day of illness. Stools are liquid on the first 2 days of illness. She was seen at an ER 2
days ago, where she was diagnosed with gastroenteritis and given ORS. She returns now
because of persistent fever. Vomiting and diarrhoea have resolved, but she is less active.
She has no cough, runny nose, or rash. Her temp is 38.9oC, heart rate 164 bpm, resp rate
48/min and peripheries are warm. She is alert and the anterior fontanelle is soft and flat.
ENT examination and lung auscultation yield no abnormalities. All other systems are also
normal. Her blood test results are as follows:
● WBC 9.4 x 109/L (5-17)
● 51% Neutrophils, 47% Lymphocytes, 2% Eosinophils
● Hb 94 g/L (115-135)
● Platelets 200 x 109/L (150-400)
10. A 7 year old boy comes for a pre-school enrolment PE. During routine questioning his
mother says he is not interested in sports or physical activities and prefers reading books.
There is no past-hx of sore throat or joint pains. On examination he is afebrile, heart rate is
96 bpm with normal pulse vol. RR is 20 breaths/min, lungs are clear. Ht and wt are at the
25th percentile. ENT examination is normal. Examination of CVS reveals a normal sized
heart with normal 1st and 2nd heart sounds. A grade 3/6 ejection systolic murmur is heard
best at the left upper parasternal area. Abdominal examination is normal. The likely
diagnosis is:
A. TOF
B. Idiopathic pulmonary arterial HTN
C. PDA
D. ASD
ASD, usually asymptomatic, can present with symptoms or complications later on; usually
during the 4th-5th decade of life. ASD → Ejection systolic murmur at left upper sternal edge
11. Shivani is a 10 year old girl whose teacher complains that she is sleepy in class and
does not pay attention. Her grades have been decreasing over the past one year. She
snores and is restless in her sleep. She has been obese since early childhood. Her present
weight is 48kg (>95th percentile) and height is 152 cm. (>95th percentile). There is no
significant past-hx or hospital admission. Systemic examination reveals no abnormalities.
The appropriate counselling for her is:
12. A 5 year old girl has fever and vomiting 7 to 8 episodes over the past 48 hours and
altered sensorium for the last 24 hours. On assessment, the growth and the development
are appropriate for age. Her heart rate is 168/min, RR is 61/min, BP is 77/38 mmHg. On PE,
she has sunken eyes and dry tongue. There is tachypnoea but lungs are clear. CNS
examination shows she is drowsy (responds to pain) and hypotonia present in all limbs.
Pupils are equal and react to light. Abdomen is distended. The Ix that will establish the dx is:
14. A 8 months old boy has recurrent urinary retention with straining during micturition and a
poor urinary stream. He is otherwise asymptomatic. The ix of choice is:
A. USG KUB
B. X-ray KUB
C. CT scan of KUB
D. Micturating cystourethrogram (MCUG)
15. Omar is a 22 months old boy referred to your clinic due to parental concerns about
speech and language delay. He was born at term by normal vaginal delivery. He is his
parent’s 4th child. Their 3 older children are all well. He crawled at 13 months and could walk
independently at 20 months. He likes to throw objects and can pick small objects up with a
mature pincer grasp. His first recognizable word was ‘dada’ at 14 months, and now he can
speak 3 clear words and no 2-word phrases. His parents have never had any concerns
about his hearing.
On examination, Omar has subtle dysmorphic features and polydactyly in both hands. Rest
of the systemic examination is normal. His wt, ht, and head circumference are all on the 50th
centile for age. The most appropriate dx for Omar is:
Doc 4
1. A 4 year old boy presented with right knee hemarthrosis. There was no past history of
surgery or any bleeding previously. The diagnosis of haemophilia A is probable with the
following family history:
3. A young mother brought her only son who is 10 years old for “not growing”. The
radiograph of his wrist showed his bone age to be about 9.5 years. The most useful way to
predict his adult height is by plotting his:
4. A 13 year old girl was diagnosed to have DM about a year ago. She is on treatment with
insulin which she injects herself. Her record of adherence has been good in the past. She
was brought into the casualty department one afternoon by her parents who noted her to be
unusually drowsy and confused. She was afebrile and did not have Kussmaul breathing. The
most useful test you want to do immediately in the casualty department is
7. As per the national immunisation schedule of malaysia, vaccination for measles in the
state of sabah is given at:
A. 6 months
B. 6 and 12 months
C. 6,9,15 months
D. 6,9,12 months and 7 years
8. a 3 month old baby girl is brought to the hospital for a runny nose. Her FBC shows Hb of
80 gm/L. WBC count and platelet count are normal. She was born full term by a normal
delivery, birth weight was 3.4kg. She had an uneventful neonatal period and is on complete
breastfeeding. There is no family history of thalassaemia. No abnormalities are found on PE.
The most likely dx is:
9. A 5 year old child recently moved from Kuantan to KL with his family. In Kuantan, he had
been on regular follow up in the paediatric clinic for the last 3 years. On examination in your
clinic, he was pale with very mild jaundice. His traube’s space was dull on percussion. No
palpable liver or spleen. His Hb level was 78 hm/L and full blood film showed hypochromic
microcytic anaemia. Which of the following features in the history would support the
diagnosis of HbH disease?
11. A 7 year old boy is on follow up for bronchial asthma. He has always had exercise
induced cough which prevents him from participating actively in his schools’ physical
activities. What advice would you give him?
Prevention of exercise induced: take b2 agonist prior to strenuous exercise is helpful (paeds
protocol)
12. A 3 year old unimmunised girl presented to the emergency department with fever, stridor
and breathlessness. A provisional diagnosis of acute epiglottitis is made. What would you do
in the emergency department to confirm this diagnosis?
13. A 4 year old presented with fever and progressive pallor. Hepatosplenomegaly was
noted on examination of his abdomen. Which of the following results will confirm the
diagnosis of acute lymphoblastic leukaemia?
15. A 5 year old child has prolonged fever associated with night sweats. He also has a
cough which occasionally produces whitish sputum. Chest x-ray was normal, and a mantoux
test was done. Which is the correct statement regarding the test?
Doc 5
1. A mother complains she was awakened in the night by her 4-year-old boy who developed
noisy breathing on inspiration, marked retractions of the chest wall, flaring of his nostril and
leans forward with his hands on his knees. He has had a mild chest infection for the past 2
days. The most likely diagnosis is :
A. Epiglottitis
B. Bronchiolitis
C. viral croup
D. foreign body in the right main bronchus
2. An 8-month-old male infant Ali is brought to the hospital emergency room by his mother.
He vomited several times that day and urine output is decreased. 3 days ago, he was
diagnosed with otitis media and given amoxicillin orally. On examination, his temperature is
40C, pulse is 80 bpm, respiration is 40/min and is regular, BP is 65/50 mmHg. His weight is
8kg. Both tympanic membranes are red and bulging, and his throat is red with enlarged
tonsils. He is lethargic and responds only to painful stimuli.
A. Lumbar puncture
B. IV Amoxicillin
C. CT scan of brain
D. Cranial ultrasound of the brain
4. A 2-month-old girl is seen in the ED for poor feeding and breathing difficulty. She has
URTI for the last 3 days but has been given no medications. Examination is significant for
a heart rate of 195 bpm, respiratory rate of 80 /min, and a BP of 60/42 mmHg. She is
irritable, skin is mottled(aka livedo reticularis), mucous membranes are dry, and CRT is 3
seconds. Chest examination shows significant intercostal retractions and crepitations are
heard in both lung fields. Heart sounds are soft. The chest x-ray shows cardiomegaly. The
likely diagnosis is:
A. Myocarditis
B. Infective endocarditis
C. Pericarditis
D. Aberrant left coronary artery
5. A 6-month-old boy was seen with fever for the past 3 days associated with vomiting
episodes and poor oral intake. On arrival to the ED, his temperature was recorded as 38.3C,
heart rate was 137 bpm, and respiratory rate was 28 /min. Examination showed sunken
eyes, a CRT of 3-4 seconds, and dry mucous membranes. Urinalysis was done which
showed specific gravity of 1.020, pH 5.5, glucose negative, protein trace, blood negative,
leukocyte esterase positive and nitrite negative. Catheterized sample urine culture showed
50,000-100,000 colony forming units (cfu)/ml of E.coli.
What is the most suitable antibiotic for the management of this child?
6. A 16-year-old boy presents with fever for the past 4 days, associated with malaise,
myalgia and frontal headache. Vital signs recorded show temperature of 37.5 C, heart rate of
102 bpm, respiratory rate of 18/min, and a BP of 112/78 mmHg. Physical exam is significant
for scrotal swelling, and tenderness on examination. Vaccination with which for the following
would have prevented this presentation?
A. MMR
B. Varicella
C. BCG
D. DTaP
Complication of mumps: orchitis +/- epididymitis ; painful swelling of 3-4x
Mumps orchitis is characterised by marked testicular swelling and severe pain accompanied by fever,
nausea, and headache. The pain and swelling resolve within 5 to 7 days, but residual testicular
tenderness can persist for weeks.
7. An 11-month-old boy was noted to be pale on his well-child checkup. Further testing
demonstrated hb 78g/L, Hct 22.9%, MCV 64 Fl, reticulocyte count of 2%(normal range
1-2%). Peripheral smear showed microcytosis, and hypochromia on smear. Which of the
following is the most appropriate management?
Pt has IDA
8. A 4-year-old boy has come to your clinic with underlying high grade vesicoureteral reflux
(VUR) and fever. He has a past history of more than 3 episodes of febrile UTI to date. He is
at risk to develop which of the following complications?
A. Hypertension
B. Increased urine output
C. Generalised edema
D. Hematuria
9. Which of the following would increase the risk of developing kernicterus in a jaundiced
neonate?
A. Metabolic alkalosis
B. Hyperalbuminaemia
C. Neonatal sepsis
D. Maternal ingestion of Phenobarbital during pregnancy (lower folic acid and vitamin K
levels, increasing the risk of spinal cord defects)
10. A 9-year-old girl presented with fever for 2 days, abdominal pain and tea coloured urine.
Her vital signs are : temperature 37.3C, pulse rate 100 bpm, respiratory rate 20/min, BP
138/90 mmHg. She appears to be irritable, otherwise no findings on physical examination.
What would you best manage this child:
Dx: AGN?
11. A 6-year-old complains of abdominal pain frequently early in the morning, for 3 months.
There is no diarrhoea, no vomiting, and she is active as usual. She is afebrile and all her
systems are normal on examination. You will investigate her for chronic abdominal pain if:
12. A 4-month-old baby is brought by her 18-year-old single mother early in the morning
saying she has accidentally scalded her baby’s right hand & forearm while she was pouring
coffee and soothing a crying baby at the same time. You will suspect child abuse if the burn:
13. Shivani is a 10 year old girl whose teacher complains that she is sleepy in class and
does not pay attention. Her grades are decreasing over the past year. She snores and is
restless in her sleep. She has been obese since early childhood. Her present weight is 48kg
(>95th percentile) and height is 152cm (>95th percentile). There is no significant past history
of hospital admission. Systemic examination reveals no abnormalities. The appropriate
counselling for her is :
A. Advise the parents on the ideal diet for the whole family
B. Investigate for Cushing syndrome
C. Advise her to use CPAP at night
D. Exclude type 1 diabetes mellitus
14. A 5-year-old girl was referred to your hospital with the symptoms of fever and vomiting
(7-8 episodes) for 48 hours and altered sensorium for the last 12 hours. Her growth and
development were appropriate for her age. Her heart rate was 168/min, respiratory rate was
61/min, and BP was 77/38mmHg. On physical examination, she had sunken eyes, dry
tongue and a wound over the left knee. Her lungs were clear, and abdomen was distended.
She was drowsy and has generalised hypotonia. Pupils were bilaterally equal and reacting
to the light. Her fundus examination was normal.
Oxygen saturation was 55%.
FBP showed neutrophilia and thrombocytosis. Blood urea nitrogen, serum calcium, serum
creatinine and serum electrolytes were within normal range. Her serum alanine
transaminase was 152 IU/L (7-55) and her test results for malarial antigen were negative.
Doc 6
1. What is the most important investigation to perform on a 3-day-old newborn baby boy who
is feeding well and thriving, but is referred for jaundice?
2. The mother of a 6 month old baby boy has come to you for advice on weaning. She has
been exclusively breastfeeding the baby until now. Your advice should include:
3. As a part of the extended program of immunisation in Malaysia, all babies would receive
hep B vaccine at birth. Which of the statements below is true regarding the vaccine?
4. Azliza is an infant who can sit forward on her hands, can reach for and grasp a cube and
transfer it from hand to hand and can babble. She can neither wave bye-bye, nor grasp
objects between her fingers and thumb. Her developmental age is most likely:
A. 10 months
B. 14 months
C. 6 months
D. 4 months
5. A 2.5 yo old presents with progressively worsening episodes of cyanosis. The child has
moments when he turns blue and becomes dyspneic. During these episodes the child
becomes irritable and remains in a squatting position. Physical examination reveals a small
and thin child with clubbing of the fingers and toes. His lungs are normal. The precordium
reveals a right ventricular lift and harsh ejection systolic murmur at the upper left sternal
border.
A. VSD
B. Tetralogy of fallot
C. Total anomalous pulmonary venous drainage
D. Transposition of great arteries
Episodes of cyanosis, cyanosis = hypercyanotic spells. Ejection systolic murmur at upper left
sternal edge = pulmonary stenosis
6. A 6 month old baby girl presents with poor feeding for 3 months. Physical examination
reveals tachypnea, tachycardia, a wide pulse pressure, a prominent apical impulse and a
continuous machinery murmur in the pulmonary area
7. A 3 year old boy presents with stridor and a barking cough. There is mild respiratory
distress, tachypnea, respiratory stridor and a temperature of 38°c.
A. Acute epiglottitis
B. Retropharyngeal abscess
C. Viral Croup
D. Bacterial Tracheitis
8. A 6 year old child presented with dark urine, facial swelling and hypertension. His skin
examination revealed 2 almost healed impetigo on the right shin.
A. IgA nephropathy
B. Post streptococcal acute glomerulonephritis
C. Henoch Schonlein Purpura joint pain, rashes
D. Minimal change nephrotic syndrome generalised edema
Hx of impetigo → AGN
9. A 6 year old boy has been diagnosed with varicella. His parents want to take him for a
holiday to france. He could be allowed to travel when:
11. A 9 months old child on exclusive breast feed is brought to the clinic for upper respiratory
tract infection and pallor. On abdominal examination, he has hepatosplenomegaly.
A. Clinically, by exclusion
B. X ray- chest, AP view
C. Throat swab culture and sensitivity
D. Blood culture and sensitivity
Repeat 13. A 6 year old boy is brought to the ED in acute respiratory distress. He is known
to be asthmatic and has a blue inhaler at home that he uses whenever he has an attack; he
has needed his inhaler a lot more than normal over the last 24 hours. On examination he
has a RR of 50/min, is using his accessory muscles of respiration and has nasal flaring. He
is able to talk only in single words. His oxygen saturation by pulse oximetry is 89%, and HR
is 140 bpm. He is started on high flow o2 in the ED and was given a salbutamol
nebulisation.
The next step in management is
14. A 5- year old boy is admitted to the paediatric ward with a sudden onset of hematuria,
edema and hypertension. He suffered from impetigo two weeks ago and he had received
three days of ampicillin at the local clinic.
15. A 9- year old boy presents with fever and joint pains. Initially the pain affected right wrist,
but now affects his left wrist and right ankle. He had tonsillitis 4 weeks ago.
On examination his heart sounds are muffled and a pansystolic murmur is heard on the left
sternal edge. His left wrist and right ankle are tender and swollen. His ESR is 95 mm/hr and
ASOT is 800 IU.
A. Septic arthritis
B. Acute rheumatic fever
C. Henoch-Schonlein disease
D. Viral myocarditis
16. A 2-month old baby boy is brought to the Emergency Department because of fever for 3
days and alteration of consciousness after a generalised tonic seizure of 5 minutes duration.
On examination he has bulging anterior fontanelle and spastic limbs.
A. Severe dehydration
B. Normal (no dehydration)
C. Mild dehydration
D. Moderate dehydration
18. A 2 year old boy presented with diarrhoea and vomiting of 3 days duration. In the last 2
days his mother gave him more concentrated formula milk. The child remained thirsty all the
time and had decreased urine output as evidenced by fewer nappies than his usual. Physical
examination revealed only mild dehydration.
What blood test results would best indicate that the child is having hyperosmolar
dehydration?
19. A 4-yr old girl was brought to the hospital following a generalised seizure of 10 minutes
duration associated with fever of 39°C. She has had one episode of febrile fits at the age of
3 yrs old. This time post Ictally she was lethargic and drowsy throughout the day.
A. Acute meningitis
B. Brain abscess
C. Complex febrile fits
D. Simple febrile fits
Physical examinations revealed mild jaundice, confined to the face only. His other systemic
examinations were normal.
Jaundice after 24 hours of life → physiological jaundice. Bilirubin levels not significantly
raised (level 1)
21. A previously healthy, 18 month old boy presents with status epilepticus. There is no past
history of family history of seizures. The child is stabilised, and the seizure stopped.
Subsequent examination reveals a playful child with a temperature of 38.8°C and nasal
discharge. Findings on the remainder of the examinations are normal.
A. Hypoglycaemia
B. Epilepsy
C. Febrile seizures.
D. Acute meningitis
Seizure associated with fever, in the absence of other causes eg. trauma, metabolic
disturbances and intracranial infections
22. BCG vaccine is given at birth and repeated if there is no scar at:
(paeds protocol)
23. A five year old previously healthy boy presented with pain in lower limbs and fever of five
days duration. On physical examination, pallor, cervical lymphadenopathy and splenomegaly
were noted.
His full blood count showed a Hb of 5.3 g/dL, leucocytes of 31.7×10^9/L and a platelet count
of 101 x 10^9/L.
24. A five year old girl came to the paediatric clinic with high fever for 6 days. On physical
examination, she had left cervical lymphadenopathy that was non-tender, mobile and
measured 4 cm, she also had non-purulent, hyperaemic conjunctiva and red inflamed
tongue, pharynx and tonsils. Her hands were swollen. State the most likely diagnosis of the
patient.
A. Measles
B. Scarlet fever
C. Kawasaki disease
D. Acute viral conjunctivitis.
Fever >5 days, +4/5 (non purulent conjunctivitis, extremities involved (oedema of hands),
adenopathy (cervical lymph node), mucosal involvement; but no rash)
25. An infant can babble and has an ulnar grasp. About two weeks ago, he started
transferring objects from one hand to the other. He should also be able to:
26. A 10 month old infant has a two day history of diarrhoea and poor fluid intake. You
diagnose him to have acute gastroenteritis with 10% dehydration.
A. Normal Saline
B. 10% dextrose.
C. Hold blood.
D. 5% D ¼ saline used for maintenance
0.9% normal saline is the most appropriate used in resus (paeds protocol)
A. Mumps
B. Infectious mononucleosis
C. Scarlet fever
D. Dengue Virus
MMR vaccine
29. A Four-year-old boy is brought to the clinic with acute onset of bleeding after a minor
injury to the foot. The bleeding continues despite application of pressure. On examination he
has multiple bruises. He develops diffuse swelling of the left elbow joint and right knee joint.
The most likely diagnosis is
30. Two-year-old child presented to the emergency room with rapid breathing. His mother
claimed that he has been having fever and cough for the last three days. On examination,
they were generalised lung crepitations, and intercostal retraction. What is the most likely
diagnosis?
A. Acute bronchiolitis
B. Foreign body inhalation
C. Immune deficiency.
D. Bronchial asthma. >5 yo
Doc 7
Repeat 1. An 18 month old boy is brought to the paediatrician because of progressively
worsening episodes of cyanosis. The child has moments where he turns blue and becomes
dyspneic. During these episodes, the child becomes irritable and remains in a squatting
position. Physical examination reveals a small and thin child with clubbing of the fingers and
toes. Lungs are normal. Heart auscultation reveals an RV lift and harsh ejection systolic
murmur at the upper left sternal border.
A. TGA
B. Total anomalous pulmonary venous drainage
C. VSD
D. ToF
2. 13 year old child developed a sudden onset of sore throat with bilateral swellings behind
his ears. His temperature was very high one week after he complained of severe epigastric
pains associated with bilateral scrotal swelling that was very painful and tender on touch.
Regarding diagnosis:
A. Acute tonsillitis
B. Diphtheritic pharyngitis
C. Diabetes Mellitus
D. Mumps
3. A mother complained that she was awakened in the night by her 2 yo son who developed
noisy breathing on inspiration, marked retractions of the chest wall, flaring of the nostrils and
a barking cough. He has had a mild respiratory infection for 2 days. The most likely
diagnosis is:
A. Epiglottitis
B. Bronchiolitis
C. Viral Croup
D. Foreign body in the right bronchus
4. What is true regarding Jones' major criteria for acute rheumatic fever?
What is the most likely factor to increase his chances of developing epilepsy - doubt D
6. Elevated levels of cholesterol and triglyceride found in a 6 yo boy whose mother reports
that he has been awakening with puffy eyes each morning. On physical examination, he has
unexpected weight gain.
A. Urine analysis
B. Echocardiography
C. Angiography of renal artery
D. Urine Culture
7. A 5 year old boy is admitted to the paediatric ward with a sudden onset of hematuria,
edema and hypertension. His mother claimed that he was having fever and sore throat 2
weeks ago and he had received three days of ampicillin at the local clinic.
8. An uncomplicated VSD in a 6-year-old boy may be associated with which one of the
following?
A. Wide and fixed splitting of the second heart sound seen in ASD normally not VSD
B. a collapsing pulse seen in aortic regurgitation
C. A holosystolic murmur of grade 4/6 in intensity
D. Clubbing of the fingers complicated
A. Scarlet fever
B. Infectious Mononucleosis
C. Measles
D. Viral C hepatitis
Repeat 10. Two year-old child presented to the emergency room with rapid breathing. His
mother claimed that he has been having fever and cough for the last 3 days. On examination
there were generalised lung crepitation and intercostal retraction.
A. Immune deficiency
B. Foreign body inhalation
C. Bronchiolitis
D. Bronchial asthma
12. A 5 year old girl developed a severe sore throat, drooling of saliva, a high fever, leaning
forwards and increasing difficulty breathing over 5 hours
A. Pneumonia
B. Viral croup
C. Acute epiglottitis
D. Bronchiolitis
A. Machinery murmur
B. Ejection systolic murmur ASD & pulmonary stenosis
C. Common in Tetralogy of Fallot VSD
D. Right to left shunt left to right
14. A 3 year old child should be able to do all of the following except
16. A 3 year old boy was admitted for the first time with pallor. There was no family history of
anaemia in the family.
Features suggesting that he has a hemolytic anaemia includes the following EXCEPT
17. A 2 year old Malay girl presented with pallor and mild hepatosplenomegaly. Her full blood
count showed the following results:
Haemoglobin : 67 g/L
White blood cell count : 11.5 x 10 9/L
Platelet count : 374 × 10° /L
Mean corpuscular volume : 65 fl (normal: 76-82 fl)
Mean corpuscular haemoglobin : 20 pg (normal: 24-30 pg)
19. A 10-year-old girl was brought in for mild fever and "reddish" rash. The rash is likely to
be petechial if it is
A. red in colour
B. mostly in the lower limbs
C. does not blanch on pressure
D. palpable in nature
20. A 20- month-old baby was brought in for fever and fits. There was no past history or
family history of febrile or afebrile fits. His development is normal. The diagnosis of
meningitis is likely ifo lol
DOC 8
a. Serum bilirubin
b. Coombs test
c. Blood group of the newborn
d. Thyroid function tests
2. The mother of a 6-month-old baby has come to you for advice on weaning.
She has been giving exclusive breast feeds to the baby until now. Your advice
should include:
4. Azliza is an infant who can sit forward on her hands, can reach for and grasp
a cube and transfer it from hand to hand and can babble. She can neither
wave bye-bye, nor grasp objects between her finger and thumb. Her
developmental age is most likely:
a. 10 month
b. 7 month
c. 4 month
d. 14 month
8. A 6 year old child presented with dark urine, facial swelling and hypertension. His
skin examination revealed 2 almost healed impetigo on the right shin.
A. IgA nephropathy
B. Post streptococcal acute glomerulonephritis
C. Henoch Schonlein Purpura
D. Minimal change nephrotic syndrome
9. A 6 year old boy has been diagnosed with varicella. His parents want to take him
for a holiday to france. He could be allowed to travel when:
11. A 9 months old child on exclusive breast feed is brought to the clinic for upper
respiratory tract infection and pallor. On abdominal examination, he has
hepatosplenomegaly.
A. Clinically, by exclusion
B. X ray- chest, AP view
C. Throat swab culture and sensitivity
D. Blood culture and sensitivity
13. A 6 year old boy is brought to the ED in acute respiratory distress. He is known
to be asthmatic and has a blue inhaler at home that he uses whenever he has an
attack; he has needed his inhaler a lot more than normal over the last 24 hours. On
examination he has a RR of 50/min, is using his accessory muscles of respiration
and has nasal flaring. He is able to talk only in single words. His oxygen saturation
by pulse oximetry is 89%, and HR is 140 bpm. He is started on high flow o2 in the
ED and was given a salbutamol nebulisation.
14. A 5- year old boy is admitted to the paediatric ward with a sudden onset of
hematuria, edema and hypertension. He suffered from impetigo two weeks ago and
he had received three days of ampicillin at the local clinic.
15. A 9- year old boy presents with fever and joint pains. Initially the pain affected
right wrist, but now affects his left wrist and right ankle. He had tonsillitis 4 weeks
ago.
On examination his heart sounds are muffled and a pansystolic murmur is heard on
the left sternal edge. His left wrist and right ankle are tender and swollen. His ESR is
95 mm/hr and ASOT is 800 IU.
A. Septic arthritis
B. Acute rheumatic fever
C. Henoch-Schonlein disease
D. Viral myocarditis
16. A 2-month old baby boy is brought to the Emergency Department because of
fever for 3 days and alteration of consciousness after a generalised tonic seizure of 5
minutes duration. On examination he has bulging anterior fontanelle and spastic
limbs.
17. A 1-year old boy is brought to Klinik Kesihatan for 3 days of fever, vomiting 3
times daily, profuse watery diarrhoea without blood and mucus and decreased urine
output. On examination he is conscious and irritable and looks very thirsty. His
weight is 9.2 kg (it was 10 kg 5 days ago). He has dry lips and tongue and his skin
turgor is less than 3 seconds.
A. Severe dehydration
B. Normal (no dehydration)
C. Mild dehydration
D. Moderate dehydration
18. A 2 year old boy presented with diarrhoea and vomiting of 3 days duration. In the
last 2 days his mother gave him more concentrated formula milk. The child remained
thirsty all the time and had decreased urine output as evidenced by fewer nappies
than his usual. Physical examination revealed only mild dehydration.
What blood test results would best indicate that the child is having hyperosmolar
dehydration?
19. A 4-yr old girl was brought to the hospital following a generalised seizure of 10
minutes duration associated with fever of 39°C. She has had one episode of febrile
fits at the age of 3 yrs old. This time post Ictally she was lethargic and drowsy
throughout the day.
A. Acute meningitis
B. Brain abscess
C. Complex febrile fits
D. Simple febrile fits
20. A 5 year old girl, known case of asthma was brought to the paediatric clinic for an
acute episode of cough and breathlessness. Which important point in the history
would suggest that this is an acute asthmatic attack?
a. History of eczema in the patient.
b. A family hx of eczema.
c. Presence of expiratory wheeze
d. Hx of passive smoking in the house.
DOC 9
1. A mother complains she was awakened in the night by her 4-year-old boy who
developed noisy breathing on inspiration, marked retractions of the chest wall, flaring
of his nostril and leans forward with his hands on his knees. He has had a mild chest
infection for the past 2 days. The most likely diagnosis is :
a. Foreign body in right bronchus
b. Acute bronchiolitis
c. Acute epiglotittis
d. Acute laryngotracheobronchitis
2. A previously healthy 2 year old girl presents with the complaint of acute-onset
wheezing. Her mother denies previous wheezing episodes. The mother says that
she left the child playing in her older brother’s room. Approximately 20 mins later,
she heard the child coughing and wheezing. Which of the following is the best next
step in management?
a. Referral to a pulmunologist.
b. Administration antibiotics for a likely pneumonia.
c. Administration of IM prednisolone.
d. Order CXR.
3. A 15 month old is playing quietly In your waiting room. The skin around the mouth
is faintly blue. but she appears comfortable. She rises from her squatting position to
run after her brother, and she suddenly becomes dyspneic and cyanotic. She returns
to a squatting position and soon is breathing comfortably with only barrier oral
sinuses. Which of the following do you expect to see on her chest radiograph?
a. Hyperinflated lungs
b. a boot-shaped heart
c. and egg on string heart
d. pulmonary plethora
5. A 6 month old infant presented with tachypnea and poor feeding. Physical
examination revealed pan systolic murmur at the left paras area over 2nd third and
fourth intercostal spaces. The baby was not cyanosed. Although his chest
auscultation revealed basal crepitation. Which of the following regarding the
diagnosis and initial treatment is true?
6. A mother complained that she was awakened in the night by her 2 yo son who
developed noisy breathing on inspiration, marked retractions of the chest wall, flaring
of the nostrils and a barking cough. He has had a mild respiratory infection for 2
days. The most likely diagnosis is:
A. Epiglottitis
B. Bronchiolitis
C. Viral Croup
D. Foreign body in the right bronchus
7. A 2-year-old child presented to the emergency room with rapid breathing. His
mother claimed that he has been having fever and cough for the last three days. On
examination they were generalized lung crepitations and intercostal retraction. What
is the most likely diagnosis?
a. bronchial asthma
b. immune deficiency.
c. Bronchopneumonia.
d. foreign body inhalation
a. Biliary atresia
b. breast milk jaundice
c. UTI
d. g6pd deficiency.
10. Mrs Ali comes with her infant who can sit with support, attempts to reach toys
out of reach, rolls over from supine to prone, but does not have pincer grasp.
mother wants to know if her infant’s developmental age is appropriate. What is his
developmental age?
a. 10 months.
b. 6months.
c. 4months.
d. 8months.
11. Mrs Hamid was screened for thalassemia. She was told she has beta
thalassemia trait, her husband's result revealed he has beta thalassemia trait as well.
She is now three months pregnant. her chance of her foetus having beta
thalassemia major is:
a. 25%
b. 50%
c. Nil
d. 75%
So, in this case, there is a 25% chance of the fetus having beta thalassemia major.
12. Mrs Chin was worried as her 15 months old son, Jason appears underweight.
She was told that Jason has failure to thrive. Jason's CW is 10.5 kg. CDC growth
chart shows the weight at 50th percentile for boys aged 15 months is 11.0 kg. you
would advise the mother that Jason:
13. Mrs Ismael says her husband had febrile fits as a child but not since. Now their
three-year-old son Rashid, has had three episodes of febrile fits within 24 hours of
onset of a febrile URTI. Each fit lasts 2 to 3 minutes. She asks your advice on
medication to prevent further recurrence of fits. You would advise:
15. A 10-month-old infant has a 2-day history of diarrhoea and poor fluid intake. You
diagnose him to be suffering from acute gastroenteritis with more than 10%
dehydration. Which of the following fluids is appropriate to begin immediate
resuscitation?
a. Whole blood.
b. 10% dextrose water.
c. normal saline.
d. 5% dextrose with 1/4 normal saline
16. Which of the following features is not associated with Down Syndrome?
a. undescended testis
b. dysmorphic features
c. VSD
d. Hypothyroidism
19. A term infant is born to a 23 year old known HIV positive mother. The mother has
been following closely during pregnancy, and she has been taking antiretroviral
medications for the weeks prior to delivery. Which of the following is indicated in the
routine management of the healthy infant?
DOC 10
1. Adam, a 5-year-old boy, is seen in the Emergency Department with a one day
history of low-grade fever, colicky abdominal pain and rash. Vital signs show
temperature of 38°C, heart rate: 100/beats per minute, respiratory rate: 20
breaths/minute. On examination, he has diffuse erythematous
macular-papular and petechial lesions on the buttocks and lower extremities.
His laboratory profile shows:
-Haemoglobin: 140 g/L
-Hematocrit 52 %
-WBC: 8 x 10%/L, segmented neutrophils: 60%, Bands: 1%, Lymphocytes: 39%
-Platelet: 160 x 10%/L
-Urinalysis: 30 red blood cells /high power field, protein 2+
-Faecal Occult Blood test: Positive
a. Bronchopneumonia
b. Foreign body aspiration
c. Bronchiolitis
d. Acute pulmonary oedema
4. A two-year old girl child was brought by her mother with complaints of
vomiting, diarrhoea and loss of appetite for the past two days. She is bottle
fed with dilute formula 90 mL every 3 hours, which she is now refusing. Her
father is a rubber tapper and mother is a housewife. She is the third child of
four children. On examination, she is lethargic and afebrile. She has skin folds
hanging loose from her axilla and thighs. Her weight is 7.5 kg (< 3rd
percentile), her heart rate is 110/min, respiratory rate is 48/min. Her capillary
refill time is 5 seconds, and she has abdominal distension. Your next course
of action should be to give
a. I.V. fluid Ringer lactate with 5% dextrose at 15 ml/kg over one hour
b. I.V Ringer lactate 20ml/kg over 30 minutes
c. I. V. normal saline 20 ml/kg over 30 minutes X
d. Formula feeds 220 ml every 2 hours.
a. Start antibiotic
b. give a fluid bolus intravenously
c. Give a second nebulization with salbutamol
d. Check his blood gas levels
6. A 45-day-old male infant was admitted with respiratory distress and crackles
all over the chest. He is afebrile, pink in room air, and has a heart rate of 166
beats/ minute. His apical impulse is palpable in the left 6th intercostal area
lateral to the midclavicular line, and liver is palpable 4 cms below the right
subcostal margin. Your management plan should be to:
9. A 5- year old boy is admitted to the paediatric ward with a sudden onset of
hematuria, edema and hypertension. He suffered from impetigo the previous
week and he had received three days of ampicillin at the local clinic. Which of
the following statement is true?
10. A 9- year old boy presents with fever and joint pains. Initially the pain affected
right wrist, but now affects his left wrist and right ankle. He had tonsillitis 4
weeks ago. On examination his heart sounds are muffled and a pansystolic
murmur on the left sternal edge. His left wrist and right ankle are tender and
swollen. His ESR is 95 mm/hr and ASOT is 800 lU, the most likely diagnosis
is:
11. A 10-month old infant has a 2-day history of diarrhea and poor fluid intake.
You diagnose a 10 % dehydration. The following fluids is appropriate to begin
immediate resuscitation:
12. A 2-month old baby boy is brought to emergency department because of fever
for 3 days and alteration of consciousness after generalized tonic seizure, for
5 minutes. On examination he has bulging anterior fontanelle and spastic
limbs. The most important investigation for diagnosis is:
13. A 1-month old baby is brought for routine vaccine schedule at klinik kesihatan.
On physical examination he is pink in color and has gained weight well. On
auscultation a machinery murmur, grade 3/6 is heard on left upper sternal
border. The most likely diagnosis is:
14. A 1-year old boy is brought to klinik kesihatan for 3 days of fever, vomiting 3
times daily and profuse watery diarrhea without blood and mucus. The boy
looks very thirsty and has reduced urine output. On physical examination his
body weight is 9.2 kg (it was 10 kg 5 days ago). He is conscious, irritable and
has dry lips and tongue. His skin turgor is less than 3 seconds. The hydration
status of the child is:
a. No dehydration
b. Severe dehydration
c. Moderate dehydration
d. Mild dehydration
15. A 7- year old boy was diagnosed to have bronchial asthma at the age of 5
years and was on treatment with steroid metered dose inhaler (MDI) daily and
MDI salbutamol to be used when necessary. He was admitted for his asthma
when response to bronchodilator was not sustained during the current attack.
On examination he has audible wheeze, mild fever and respiratory distress.
On auscultation of his chest revealed that his breath sound was equal
bilaterally and has prolonged expiratory phase. There were bilateral rhonchi,
but no crepitation was detected. His Sp02 was 95% on 2L/min of Oxygen via
nasal catheter. He would require:
16. A 3-year old boy was admitted with first episode of generalized seizures. The
diagnosis of meningitis is more likely than that of febrile seizure if
17. A 3- year-old girl presented with acute diarrhoea and vomiting of 3 days
duration. She received I.V. fluid for 3 hours. Acute kidney injury (renal failure)
as a complication is suggested if the child
18. A 1-year old baby presented with acute respiratory distress. A diagnosis of
bronchiolitis is most likely if, on examination, there is
a. subcostal recession
b. generalized rhonchi
c. generalised crepitation
d. high grade fever
a. 6, 9 and 15 months
b. 6 and 12 months
c. 6 months
d. 6, 9, 12 months and 7 years
20. Siti, a 6-year-old girl, is seen in the clinic for a 3 weeks history of low grade
fever, fatigue, weight loss, myalgia and headaches. On examination, she has
a new heart murmur, petechiae, and splenomegaly. Which of the following is
the most appropriate next step in the diagnosis of this child's condition?
a. Obtain an echocardiogram
b. Measurement of anti-streptolysin antibodies
c. Streptococcal throat culture
d. Bone marrow aspiration and examination
21. Ravi, a 4-year-old boy presented with high grade fever of 40°C for 5 days,
associated with cough, runny nose and generalized macular-papular rash
which initially started along the hairline area for the past 1 day. His
immunization history is not known. What is the most likely clinical diagnosis?
a. Rubella
b. Parvovirus B19
c. Measles
d. Varicella Zoster
22. Vera, a 5-year-old girl came to the pediatric clinic with high fever for 6 days.
On physical examination she had left cervical lymphadenopathy that was non
tender, mobile, measuring 6 cm and non-purulent hyperemic conjunctiva. She
also had red inflamed tongue, pharynx and tonsils. Her hands were swollen.
What is the most likely diagnosis?
a. Kawasaki disease
b. Acute viral conjunctivitis
c. Scarlet fever
d. Measles
23. Rina, a 7-month-old girl has high grade fever for 3 days associated with
lethargy, seizures and loss of consciousness. Examination of CSF confirms
the diagnosis of meningitis. The earliest clinical sign of meningitis in this infant
would be:
a. Nuchal rigidity
b. Babinski sign
c. Bulging fontanel
d. Tripod sign
24. The mother of a six-month-old baby has come to you for advice on weaning.
She has been giving exclusive breast feeds to the baby until now. Your advice
should include
25. Warning Signs in dengue fever include all of the following EXCEPT:
a. Abdominal pain
b. Fever
c. Persistent Vomiting
d. Lethargy
27. Brian, a 4-year-old boy was brought to the Emergency department because
mother was concerned about his worsening respiratory status. He had
symptoms of upper respiratory tract infection for the past 2 days. On
examination, his respiratory rate was 60 breaths/ minute, SpOz was 90%, and
he had noisy breathing on inspiration with marked retractions of the chest
wall. He also had flaring of the nostrils and he was leaning forward putting his
hands on his knees. What is the the most likely diagnosis?
28. A child is brought to the paediatric clinic for a well-child visit. He can support
his head but cannot sit with support. He has a social smile and laughs but
cannot transfer toys from one hand to another. What is the most likely age of
this child?
a. 10 months
b. 8 months
c. 6months
d. 4 months
30. A 6-month-old baby girl presents with poor feeding for 3 months. Physical
examination reveals tachypnea, tachycardia, a wide pulse pressure, a
prominent apical impulse and a continuous machinery murmur in the
pulmonary area. The most likely diagnosis, is
DOC 11.
1. A 3-year-old boy has come to the emergency department with the history of
fever, and generalised tonic clonic seizure, lasting for 5 minutes. Post ictally,
the patient regains consciousness and plays as usual. His father had similar
symptoms while he was at his age. On physical examination he has no nuchal
rigidity. Brudzinski sign and Babinski sign are absent. Name the next step to
do.
a. EEG
b. Lumbar puncture
c. CT SCAN
d. Reassurance to the parent
4. A 5-year old girl came to the pediatric clinic with high fever for 6 days. On
physical examination she had left cervical lymphadenopathy that was non
tender, mobile, measuring 6 cm and non-purulent hyperemic conjunctiva. She
also had red inflamed tongue, pharynx and tonsils. Her hands were swollen.
State the most likely diagnosis of the patient.
a. Measles
b. Kawasaki disease
c. Acute viral conjunctivitis
d. Scarlet fever
5. A 15-year-old girl has a two-week history of fatigue and back pain. She has
widespread bruising, pallor and tenderness over the vertebrae and both
femurs. She has a liver palpable 4 cm below the costal margin and a hard
spleen measuring 6 cm. Complete blood count shows hemoglobin
concentration of 50 g/L, leukocyte count of 2 x 10%/L, and platelet count of 15
x 10%. She should be investigated for:
a. Infectious mononucleosis
b. Meningococcemia
c. Idiopathic thrombocytopenic purpura
d. Acute Leukemia
DOC 12
1. A 28-year-old primigravida mother was brought to the hospital for fever and
leaking liquor. Caesarean section was done because of prolonged rupture of
membrane and failed induction. A normal 37 weeks gestation baby boy was
safely delivered. However about one hour after delivery he was noted to have
rapid breathing and mild respiratory distress. The following statement is true
regarding the baby respiratory problem:
2. A 5-year-old boy was brought to the clinic for headache, projectile vomiting
and unsteady gait. Which of the following signs present in him is suggestive of
the diagnosis cerebellar tumour?
a. Convergent squint
b. Bitemporal hemianopia
c. Papilledema
d. Horizontal nystagmus
5. Ali, a 4 -year-old miserable and docile looking child is brought to the clinic for
multiple bruises. The mother describes him as a hyperactive and clumsy child
and prone to injury but cannot not explain specifically how the recent big
bruise on his back come about. The mother works in a supermarket and while
is away at work, the child is looked after by Ali’s 34-year-old stepfather who is
the mother's second husband. Since the mother is going to be on leave from
work the next two days, she wants the child, who is otherwise well, to be
treated as outpatient. Your management of the patient include all of the
following EXCEPT "(DAMA: Discharge against medical advise)
6. A 3-year-old child was brought to the clinic for abnormal skin lesions of recent
onset. Which of the following findings could help in the respective diagnosis?
a. Laryngomalacia
b. Foreign body obstruction of airway
c. Acute laryngotracheobronchitis
d. Acute epiglottitis
a. Nebulized beta-agonist
b. Parenteral antibiotics
c. Humidified oxygen
d. Oral antibiotics
10. A 6-year-old girl presents with multiple pin-point non-blanching rashes over
her legs. She has low grade fever with sore throat 3 weeks ago. Physical
examination shows no abnormal findings except petechiae over the legs.
There is no mucosal bleeding. Initial full blood count shows - ITP observe first
Haemoglobin 115 g/L
wcc 6.5 x 109/L
Platelet 40 X 109/L
Peripheral smear large platelets, other cells - normal
What is the most appropriate management?
11. A 3-year-old boy presents with facial puffiness and abdominal distension.
Urine output is normal and his blood pressure is normal. Investigation results
show normal complement levels and normal renal function. A diagnosis of
nephrotic syndrome is made after investigations. Which of the following
statements is true regarding management of this boy?
12. A 10-month-old boy presented with fever. The diagnosis of recurrent urinary
tract infection was made and planned to do imaging studies to detect
underlying anatomical abnormalities.
What is the best imaging study to detect vesicoureteral reflux?
a. Moderate dehydration
b. No dehydration
c. Mild dehydration
d. Severe dehydration
17. A 13-year-old boy is brought to the hospital with complaints of headache and
vomiting. He is drowsy but oriented and afebrile. His pulse rate is 64/minute,
respiratory rate of 30 minute and his blood pressure 140/100 mmHg. He was
treated for skin infection 3 weeks ago. Had passed less urine over the last 24
hours. The investigation that will be most relevant to arrive at a diagnosis is
a. CT scan brain
b. ASO titre
c. Serum electrolyte
d. Urinalysis
20. A 2-month-old infant is noted to have a cardiac murmur. All the pulses are
normal, the oxygen saturation is 70% under the room air and does not
significantly improved with oxygen therapy. Which is the most likely of the
following congenital heart defect?
21. Which of the following conditions is associated with early heart failure?
22. At day 1 of life premature neonate with birth weight of 1500g was admitted to
the NICU for respiratory distress syndrome and was ventilated. However, at
day 5 of life he was noted to have a systolic murmur grade 3/6 at the upper
left sternal border with bounding pulse. He is pink and on auscultation the
lungs are clear.
What is the most likely diagnosis?
a. Ventricular septal defect
b. Tetralogy of Fallot
c. Transposition of great arteries
d. Patent ductus arteriosus
23. Which of the following conditions is associated with early heart failure?
24. A 16-year-old girl is referred to the paediatric endocrinology clinic for delayed
puberty. On examination her height is below the 3"‘ centile and she has an
ejection systolic murmur. Her gonadotrophin level is high. What is the most
likely diagnosis?
a. Marfan syndrome
b. Down syndrome
c. Turner syndrome
d. Hypothyroidism
25. A baby was bom with bilateral cataract, patent ductus arteriosus and
microcephaly. The most likely diagnosis is
26. A 6-year-old boy was brought to the Paediatric Neurology Clinic for follow up
of his diplegia. The following clinical finding would suggest that the diplegia is
due to an upper motor neuron lesion:
a. Babinski reflex
b. Pendular knee jerks (reflexes)
c. Hypotonia of the lower limbs
d. Horizontal nystagmus
27. A 7-year-old boy had fever, jaundice, vomiting and abdominal pain. Two
weeks
ago, he and his family had consumed half-cooked seafood.
On physical examination, he looked ill and was febrile ( 38° C). He had mild
jaundice and his liver was tender and was palpable 4 cm below the right
costal
margin.
The following investigation would confirm the diagnosis
29. A 16-year-old boy presented with fever for the past 4 days, associated with
malaise, myalgia and frontal headache. Vital signs recorded showed
temperature of of 37.50 C, heart rate 102 bpm, respiratory rate 18 breaths per
minute, and blood pressure 112/78 mmHg. The only other significant finding
on examination was
bilateral tender scrotal swelling.
a. Varicella Zoster
b. Measles- Mumps- Rubella
c. Diptheria- Tetanus-Pertussis
d. Bacillus Calmette Guerin (BCG)
a. Duodenal atresia
b. Large anterior fontanel
c. Ataxic gait
d. Hypothyroidism
DOC 13.
1. A 3-year-old baby was brought to the emergency department because fever and
runny nose for 2 days and the first episode of generalised clonic seizures of less
than 5 minutes duration with up-rolling of eyes on the day of admission.
Post-ictally, the baby cried and fed normally and remained active. Her father had
history of seizures during his childhood.
On physical examination, the child was alert and well; Babinsky and Brudzinski
signs were negative and there was no paralysis.
What is the most likely diagnosis in this child?
A. Viral encephalitis
B. Grand mal epilepsy
C. Complex febrile seizure
D. Simple febrile seizure
3. A 9-year-old boy presented with 5 days fever and joint pain. Initially, the pain
affected his right wrist, but on the third day, he had pain in his left and right ankle.
He had history of tonsilitis 4 weeks ago.
On examination he was febrile but not in respirator distress. There was no pallor
or ankle oedema, and all the peripheral pulses were normal. The heart sounds
were soft, and a systolic murmur was heard on the left lower sternal edge. His
left and right ankle were red, warm, tender, and swollen.
His ESR was 95 mm/hr (normal <20 mm/hr) and ASOT 800 IU/L (normal 333
IU/L). State the most likely diagnosis.
A. Septic arthritis
B. Acute rheumatic fever
C. Henoch-Schoenlein vasculitis
D. Viral myocarditis
6. A 6-week-old baby was diagnosed to the health clinic for vaccination. On routine
examination, he was noted to have a large tongue, umbilical protrusion, a large
anterior fontanel, and mild jaundice. On questioning, his mother said he passed
stools once in 3-4 days.
The baby should be tested for:
A. Serum iron and ferritin
B. Serum calcium level
C. Glucose-6-phospgate dehydrogenase level
D. Thyroid function test
10. A baby was born with bilateral cataract, patent ductus arteriosus and
microcephaly.
The most likely diagnosis is:
A. Maternal alcohol syndrome
B. Osteogenesis imperfecta
C. Congenital rubella syndrome
D. Turner syndrome
11. A 6-year-old boy was brought to the paediatric neurology clinic for follow-up of
this diplegia.
The following clinical finding would suggest that the diplegia is due to an upper
motor neuron lesion:
A. Babinski reflex
B. Pendular knee jerks (reflexes)
C. Hypotonia of the lower limbs
D. Horizonal nystagmus
12. A 5-year-old girl was brought to the paediatric clinic for high fever of 6 days. On
physical examination she had left cervical lymphadenopathy that was
non-tender, mobile, measuring 6 cm in diameter and non-purulent hyperaemic
conjunctiva. She also had red inflamed tongue, pharynx, and tonsils. Her hands
were swollen.
The most important investigation is:
A. Echocardiography
B. Blood culture and sensitivity
C. Erythrocyte sedimentation rate
D. Full blood count
13. A baby boy was delivered by a 25-year-old mother via and elective Caesarean
section. The baby appeared well and was active with no respiratory distress. The
laboratory investigations of the mother showed the following hepatitis B serology
results: HBsAg positive, anti-HBc positive, and anti-HBs negative. The
appropriate baby feeding advice is:
A. Formula milk
B. Dependant on the baby’s hepatitis B serology result
C. Milk from a milk bank
D. Breastfeeding
14. A 7-year-old boy had fever, jaundice, vomitting and abdominal pain. Two weeks
ago, he and his family had consumed half-cooked seafood.
On physical examination, he looked ill and was febrile (38°C). He had mild
jaundice and his liver was tender and was palpable 4 cm below the right costal
margin.
The following investigation would confirm the diagnosis:
A. IgM anti-HAV
B. Anti-HBs
C. HBsAg
D. IgG anti-HCV
15. A mother complaint that she was awakened in the night her 4-year-old boy who
developed noisy breathing on inspiration, marked retractions of the chest wall,
flaring of his nostrils. He woke up and sat on the bed leaning forward with his
hands on his knees and with saliva drooling from his mouth. He has a mild chest
infection for the past 2 days.
The most likely diagnosis is:
A. Acute epiglottitis
B. Acute bronchiolitis
C. Foreign body in the right main bronchus
D. Acute laryngotracheobronchitis (croup)
16. The following statement is true regarding respiratory distress syndrome of the
newborn (hyaline membrane disease):
A. Commonly starts within a few hours after birth.
B. It is caused by aspiration of meconium.
C. X-ray shows unilateral lung opacity.
D. Expiratory grunt is a characteristic clinical feature.
17. A 16-year-old boy presented with fever for the past 4 days, associated with
malaise, myalgia, and frontal headache. Vital signs recorded showed
temperature of 37.5°C, heart rate 102 beats/min, respiratory rate eighteen
breaths/min, and blood pressure 112/78 mmHg. The only other significant finding
on examination was bilateral tender scrotal swelling.
Which childhood vaccination could have prevented this presentation? SI THU
A. Diphtheria-Tetanus-Pertussis (DTaP)
B. Bacillus Calmette-Guerin (BCG)
C. Measles-Mumps-Rubella (MMR)
D. Varicella Zoster
18. Which of the following conditions would increase the risk of developing
kernicterus in a jaundiced neonate?
A. Hypoalbuminaemia
B. Metabolic alkalosis
C. Maternal ingestion of phenobarbitone during pregnancy
D. Neonatal sepsis
20. A 6-year-old boy with acute lymphoblastic leukaemia in the induction phase of
treatment had fever, cough, and runny nose since this morning. His brother had
fever and runny nose as well.
On physical examination, his body temperature was 38°C. He was alert and had
no respiratory distress. On auscultation of the lungs, vesicular breath sound with
prolonged expiration was noted.
Haemoglobin was 110 g/L, WBC 1.5 x 103/μL, and platelet 150 x 103/μL.
The appropriate immediate management is:
A. Reassure the parent that it is nothing serious.
B. Do a bone marrow examination.
C. Give anti-pyretic and plenty of fluids.
D. Start empiric intravenous antibiotic.
21. A 6-year-old boy was brought to the clinic for pallor, fever, and gum bleeding for
1 week. On physical examination, his temperature as 38°C; his liver and spleen
were enlarged.
Hb was 80 g/L; WBC 20 x 109/L; ( high) platelet count 90 x 109/L. ( low )
The investigation that would confirm the diagnosis is:
A. Factor VIII assay
B. Blood film morphology
C. Dengue serology
D. Bone marrow examination
24. A 9-year-old boy was preterm at 32 weeks and had birth history of hypoxia. He
was immediately admitted to the NICU where he was ventilated for 4 weeks. His
brain CT scan done while he was in NICU showed the presence of
periventricular haemorrhage. Subsequently, the patient was diagnosed with
cerebral palsy.
He currently goes to a special school and walk with an abnormal gait.
State the most likely gait seen in this patient.
A. High stepping gait
B. Waddling gait
C. Spastic diplegic gait
D. Broad-based gait
26. A 2-year-old child developed swelling of his eyes and ankles over the past week.
Blood pressure was 100/60 mmHg, pulse 110 beats/min, and respiratory rate 28
breaths/min. In addition to periorbital swelling and pitting oedema of his ankles,
he also had abdominal distention with a positive shifting dullness.
His blood test results showed serum creatinine of 0.4 mg/dL, albumin 1.4 g/dL,
and cholesterol 569 mg/dL. His urinalysis showed heavy proteinuria without
haematuria.
The most likely diagnosis is:
A. Henoch-Schoenlein vasculitis with nephritis
B. Acute post-streptococcal glomerulonephritis
C. Haemolytic-uraemic syndrome
D. Minimal change nephrotic syndrome
27. It is important for medical staff to have good communication skill. Communication
with patients/their parents should have the following characteristics EXCEPT:
A. It should be sensitive to patient’s cultural background.
B. Patient/parents allowed to be involved in determining medical care.
C. Non-verbal is not as important as verbal communication.
D. It should be patient oriented.
28. A 6-year-old boy brought into the A&E department in acute respiratory distress.
He was known to be asthmatic. His mother informed that he had needed his
inhaler a lot more times than normal over the past 24 hours. On examination, he
was tachypnoeic with a respiratory rate of 50 breaths/min. He used his
accessory muscles of respiration and has an audible wheeze. He was able to
talk only in single word sentence.
On arrival in the A&E, his oxygen saturation was 85% and his heart rate was 160
beats/min. He was started on high flow oxygen and had already been nebulised
once with salbutamol without much improvement.
The next step in management should be:
A. Give a second salbutamol nebulization.
B. Give a normal saline bolus infusion.
C. Give steroid nebulization.
D. Check his arterial blood gas levels.
29. A 6-month-old baby bot was born by spontaneous vaginal delivery at 38 weeks
of gestation after an uncomplicated pregnancy and was doing well. He had
undergone circumcision yesterday and this morning, blood was seen to still be
oozing from the circumcision site. His maternal uncle had a bleeding problem for
which he was medication three times every week. There was no significant
history of illness on his paternal side.
Name the type of inheritance in this disease.
A. X-linked recessive
B. Autosomal dominant
C. DNA-linked
D. Autosomal recessive
30. A 3-month-old baby boy was brought to the health clinic for immunization.
According to his mother, the baby fed well on the breast and was already active.
His weight and height were on the 50th percentile.
On examination, the infant was pink; his pulse was non-collapsing and regular,
with a rate of 140 beats/min. His respiratory rate was 40 breaths/min. On
auscultation of the chest, there was a holosystolic murmur, grade 4/6, in the left
lower parasternal area.
The most likely diagnosis is:
A. Patent ductus arteriosus
B. Large ventricular septal defect
C. Transposition of the great arteries
D. Small ventricular septal defect
DOC 15
1) A 28-year-old primigravida mother was brought to the hospital for fever and leaking liquor.
Caesarian section was done because of prolonged rupture of membrane and failed induction. A
normal 37 weeks gestation baby boy was safely delivered. However about one hour after delivery he
was noted to have rapid breathing and mild respiratory distress. The following statement is true
regarding the baby respiratory problem:
Select one:
a. Transient tachypnoea of the newborn should be the topmost diagnosis in this case.
b. The baby should be treated with antibiotics for early neonatal (congenital) pneumonia.
c. The mother should have been given steroids to prevent respiratory distress.
d. Meconium aspiration syndrome is the diagnosis if the liquor is meconium stained.
2) A 3-month-old baby boy was brought to the Health Clinic for immunisation. According to his
mother, the baby fed well on the breast and was always active. His weight and heights were on the
50th percentile. On examination, the infant was pink; his pulse was non collapsing and regular, with a
rate of 140 beats per minute. His respiratory rate was 40 breaths per minute. On auscultation of the
chest, there was a holosystolic murmur, grade 4/6, in the left lower parasternal area. The most likely
diagnosis is:
Select one:
a. Small ventricular septal defect the smaller the defect, the louder the murmur
b. Large ventricular septal defect
c. Transposition of the great arteries
d. Patent ductus arteriosus
3) A 4-year-old baby is brought to the emergency department because of fever and one
episode of generalized clonic seizure, less than 5 minutes duration with up rolling eye ball. Postictally
the baby cried, was active and has no paralysis. The father has history of seizures during his
childhood. On examination the child is active; Babinsky and Brudzinsky signs are negative and no
paralysis is detected. What is the most likely diagnosis?
Select one:
a. Complex febrile seizures focal, prolonged (>15 mins), >1 attack in 24 hrs
b. Grand mal epilepsy generalised tonic clonic seizure
c. Viral encephalitis
d. Simple febrile seizures
4) A 3-year-old boy presents with facial puffiness and abdominal distension. Urine output is
normal and his blood pressure is normal. Investigation results show normal complement levels and
normal renal function. A diagnosis of nephrotic syndrome is made after investigations. Which of the
following statements is true regarding management of this boy?
Select one:
a. Oral diuretics are the first line of management.
b. Renal biopsy is indicated.
c. Immediate referral to pediatric nephrologist.
d. Oral corticosteroid is the treatment. Unless there are atypical features. Oral prednisolone is given
5 ) A 5-month-old infant has high fever, lethargy, seizures and loss of consciousness.
Examination of CSF confirms the diagnosis of meningitis. The earliest clinical sign of meningitis in this
infant would be:
Select one:
a. Bulging fontanel
b. Tripod sign
c. Nuchal rigidity
d. Babinski sign
6) A 16-year-old boy presented with fever for the past 4 days, associated with malaise,
myalgia and frontal headache. Vital signs recorded showed temperature of 37.50 C, heart
rate 102 bpm, respiratory rate 18 breaths per minute, and BP is 112/78 mmHg. The only other
significant finding on examination was bilateral tender scrotal swelling. Which childhood
vaccination could have prevented this presentation?
Select one:
a. Varicella Zoster
b. Measles-Mumps-Rubella (MMR)
c. Bacillus Calmette-Guerin (BCG)
d. Diphtheria-Tetanus-Pertussis (DaP)
7) A 2-month-old infant is noted to have a cardiac murmur. All the pulses are normal; the
oxygen saturation is 70% under the room air and does not significantly improve with oxygen therapy.
Select one:
a. Tetralogy of Fallot
b. Ventricular septal defect
c. Patent ductus arteriosus
d. Atrial septal defect
8) A 13-year-old boy presented to his primary care provider with a 2-day history of abdominal
pain, diarrhea and vomiting. The abdominal pain was sharp, originating in the epigastric region and
exacerbated by movement. There was history of eating outside food. On examination he was febrile,
tired looking with no signs of dehydration. There was tenderness in the epigastrium and in the right
lower quadrant of abdomen on deep palpation. The most likely diagnosis is:
Select one:
a. Acute cholecystitis
b. Acute appendicitis
c. Food poisoning more severe, likely to have signs of dehydration
d. Acute gastroenteritis
9) A 10-month-old boy presented with a fever. The diagnosis of recurrent urinary tract
infection was made and planned to do imaging studies to detect underlying anatomical abnormalities.
What is the best imaging study to detect vesicoureteral reflux?
Select one:
a. DMSA scan
b. Plain abdominal X-ray
c. Ultrasound of the abdomen and pelvis
d. Micturating cystourethrogram
10) A 12-month-old baby boy was brought to a klinik kesihatan because of profuse watery
diarrhea for 3 days. The baby looked very thirsty and drank eagerly though he subsequently vomited.
His weight was 9.3 kg (it was noted to be 10 kg three days prior to the diarrhea). He had passed
adequate urine less than 3 hours ago. On examination anterior fontanel was sunken, dry oral mucosa
and skin turgor less than 2 seconds. What is the severity of his dehydration?
Select one:
a. Severe dehydration
b. No dehydration
c. Moderate dehydration if skin turgor > 2 seconds then its severe
d. Mild dehydration
11) Fiza, a 3-year-old girl, was admitted to the hospital for severe dehydration and
shock due to acute gastroenteritis. She had no urine output for the whole morning
and her ECG showed peaked T-wave. What is the likely cause of this ECG
change?
Select one:
a. Hypernatremia
b. Hyperkalemia
c. Hypokalemia
d. Hyponatremia
12) It is important for medical staff to have good communication skills. Communication with
patients/their parents should have the following characteristics EXCEPT
Select one:
a. Nonverbal is not as important as verbal communication
b. It should be sensitive to patient's cultural background
c. Patient/parents allowed to be involved in determining medical care
d. It should be patient orientated
13) An 8-year-old boy came to klinik kesihatan with the following hepatitis B serology results:
positive HBsAg; positive IgM anti HBc and negative anti-HBs. Interpret the serology result.
Select one:
a. He is immune to Hepatitis B due to vaccination
b. No infection
c. He has chronic Hepatitis B infection
d. He has acute Hepatitis B infection
14) Ann, a 14-year-old girl, saw a private gynaecologist for insertion of IUCD for
contraception. She had just entered college and lived with her loving parents. However, her visit to the
clinic was accompanied by her boyfriend. The parents did not know of their promiscuous relationship.
The doctor correctly refused to carry out the procedure based on the following ethical principle
Select one:
a. Beneficence
b. Autonomy
c. Non maleficence
d. Justice
16) Ali, an 8-month-old infant was brought to the hospital for progressive pallor. Both his
parents were beta thalassaemia carriers.On examination, Ali had severe pallor and
hepatosplenomegaly. His Hb level was 65 g/L with low mean corpuscular volume (MCV) and low
mean corpuscular haemoglobin (MCH). His white blood cell and platelet counts were normal. Which
of the following statements is true regarding his condition?
Select one:
a. Hb electrophoresis need to be done to diagnose thalassaemia major.
b. From the family history, Ali has 50% chance of having homozygous beta thalassaemia.
c. Acute lymphoblastic leukaemia is a likely diagnosis. Thrombocytopenia present, fever & petechiae
present
d. The most likely diagnosis is iron deficiency anaemia. Won’t have hepatosplenomegaly usually
17) All newborns in Malaysia are screened for congenital hypothyroidism under the National
Screening Program. The rationale is to enable early diagnosis and start thyroxin replacement therapy
early to:
Select one:
a, reduce the incidence of jaundice and hence of kericterus.
b. prevent patient from getting hypothermia during nedsatal period.
c. prevent mental retardation which is not reversible with late treatment.
d. prevent constipation associated with hypothyroidism.
18) An 8-month-old baby came to emergency department with fever, rapid breathing
associated with runny nose for 3 days. On examination respiratory rate was 52 breaths per minutes;
there was clear nasal discharge and suprasternal and intercostal recession were seen. Prolonged
expiration and rhonchi were appreciated almost all over the chest. Chest X-ray showed hyperinflated
lungs. Name the most common pathogenic agent that has caused the disease.
Select one:
a. Influenza virus
b. Echovirus
c. Respiratory syncytial virus (Acute bronchiolitis)
d. Adenovirus
19) A 6-month-old baby boy was born by spontaneous vaginal delivery at 38 weeks of
gestation after an uncomplicated pregnancy and was doing well. He had undergone circumcision
yesterday and this morning blood was seen to still be oozing from the circumcision site. His maternal
uncle had a bleeding problem for which he was on medication three times every week. There was no
significant history of illness on his paternal side. Name the type of inheritance in this disease.
Select one:
a. DNA linked
b. X-linked recessive haemophilia
c. Autosomal recessive
d. Autosomal dominant
20) A 9-year-old boy is admitted to the hospital for multiple joint pain and tenderness for the
period of 2 weeks. He also gave a history of fever with cough. The throat was inspected and the
tonsils were enlarged. Which of the investigations is most useful to confirm the diagnosis?
Select one:
b. ESR
c. ECG
22) A previously well 7-month-old girl presents with low grade fever and noisy breathing for 2
days. It is preceded by coryzal symptoms for 2 days. Physical examination reveals mild respiratory
distress, hyperinflation of the chest and generalized high-pitched wheezes. What is the initial most
appropriate management for this girl?
Select one:
a. Humidified oxygen
b. Parenteral antibiotics
c. Nebulized beta-agonist
d. Oral antibiotics
Ans: oral or IM dexamethasone is usually given
23) A 9-year-old boy was born preterm at 32 weeks and had birth history of hypoxia. He was
immediately admitted to the NICU where he was ventilated for 4 weeks. His brain CT scan done while
he was in NICU showed the presence of periventricular haemorrhage. Subsequently the patient was
diagnosed to have cerebral palsy. He currently goes to a special school and walk with an abnormal
gait. State the most likely gait seen in this patient is
Select one:
a. waddling gait DMD
b. high stepping gait foot drop (peroneal nerve injury)
c. spastic diplegic gait
d. broad-based gait cerebellar lesion
24) A baby was born with bilateral cataract, patent ductus arteriosus and microcephaly. The
most likely diagnosis is
Select one:
a. Turner syndrome
b. osteogenesis imperfecta
c. maternal alcohol syndrome
d. congenital rubella syndrome
25) A 6-year-old non-immunized aboriginal boy presented to the Paediatric Clinic for the first
time. When asked why he had not been immunised, his mother replied she "saw on TV that vaccines
can hurt babies". Which of the following vaccine combinations would you administer on this visit?
Select one:
a. HBV, DT, MMR, Hib
b. HBV, DTaP, IPV, Varicella
c. Hepatitis B (HBV), DTaP, MMR & Hib
d. HBV, DT, IPV, MMR
DOC 16
1) (Repeat of doc 15)
3) A 2 year old child developed swelling of his eyes and ankles over the past week. Blood pressure
was 100/60 mm Hg, pulse 110/min, respiration 28/min. In addition to periorbital swelling and pitting
oedema of his ankles, he also had abdominal distention with a positive shifting dullness. His blood
test results showed serum creatinine of 0.4 mg/dL, albumin 1.4 g/dL and cholesterol 569 mg/dL. His
urinalysis showed heavy proteinuria without haematuria. The most likely diagnosis is:
A) Acute poststreptococcal glomerulonephritis.
B) Henoch-Schonlein Vasculitis with nephritis.
C) Minimal Change Nephrotic Syndrome.
D) Haemolytic-Uraemic Syndrome.
7) A 5 year old girl was brought to the Paediatric Clinic for a high fever for 6 days. On physical
examination, she had left cervical lymphadenopathy that was non-tender, mobile, measuring 6 cm in
diameter and non-purulent hyperemic conjunctiva. She also had a red inflamed tongue, pharynx and
tonsils. Her hands were swollen. The most important investigation to be done is:
A) Blood Culture and Sensitivity.
B) Erythrocyte Sedimentation Rate.
C) Full Blood Count.
D) Echocardiography.
Ans: The clinical features are suggestive of Kawasaki disease. Can lead to coronary artery
aneurysms, and early detection through echocardiography is crucial for appropriate management and
prevention of potential complications.
9) A 6 year old boy with acute lymphoblastic leukaemia in the induction phase of treatment has had
fever, cough and runny nose since this morning. His brother had a fever and runny nose as well. On
PHysical examination, the body temperature was 38°C. He was alert and had no respiratory distress.
On auscultation of the lungs, vesicular breath sounds with prolonged expiration were noted. Hb was
110g/L, WBC 1.5x103u/l and Platelet 150x103 u/L. The appropriate immediate management is:
A) Done a bone marrow examination.
B) Start empiric intravenous antibiotics.
C) Reassure the parents that it is nothing serious.
D) Give antipyretics and plenty of fluids.
11) A mother complained that she was awakened in the night by her 4 year old boy who developed
noisy breathing on inspiration, had marked retractions of the chest wall and flaring of his nostrils. He
woke up and sat on the bed leaning forward with his hands on his knees and with saliva drooling from
his mouth. He has had a mild chest infection for the past 2 days. The most likely diagnosis is:
A) Acute Epiglottitis. Tripod position due to acute upper airway obstruction
B) Foreign Body in the Right Main Bronchus.
C) Acute Bronchiolitis.
D) Acute Laryngotracheobronchitis (Croup).
21) John, a 15 month old boy needed intravenous fluid for maintenance of daily requirements. His
body weight is 12 kg. What is his fluid requirement for daily maintenance?
A) 1200 ml/24 hour.
B) 240 ml/24 hour.
C) 1100 ml/24 hour.
D) 120 ml/24 hour.
Ans: Holliday-Segar method
22) Nurul, a 5 year old girl presented to the Emergency Department for fever for 3 days and pin-point
non-blanching rashes on the body. She also complained of aches and pain. Initial full blood count
showed leukopenia. What is the most probable diagnosis?
A) Dengue. Fever, aches, pain, rashes present but are mostly maculopapular rashes
B) Rubella. No mention of lymphadenopathy or cold-like symptoms
C) Measles. Fever, koplik spots, runny nose, conjunctivitis, rashes
D) Meningococcaemia. Pin-point non-blanching rashes, leukopenia, fever, aches, pain
23) Puan Aminah and Encik Awang brought their 12 year old adopted son who they felt was not
growing tall and fast enough as his classmates. Before counselling the couple, you would do all the
following EXCEPT:
A) Take the midparental height of his real parents. Weird to contact real parents and measure them
idk
B) Do an X-Ray of his phalanges to assess bone age. Commonly done
C) Measure the upper and lower segment of his height.
D) Take a detailed pubertal assessment.
24) The height-for-age of Rokeshwar, a 10 month old immigrant boy was found to be below the 3rd
centile. He is dark skinned and physical examination shows swollen wrist, palpable costochondral
junctions and Harrison's sulcus. What is the most likely cause of his clinical manifestations?
A) Cushing Syndrome.
B) Chronic Renal Failure.
C) Vitamin D deficiency. Rickets symptoms r seen
D) Achondroplasia.
25) The mother of Ahmad, a 10 month old boy, a known case of Tetralogy of Fallot, called over the
phone and said that Ahmad was having a more intense cyanosis while crying. Which of the following
advice should be given to the mother to relieve the cyanosis as the first step?
A) Do tepid sponging.
B) Put in the left lateral position.
C) Bring knees to the chest.
D) Note the time.
Ans: Knee-chest position can temporarily increase systemic vascular resistance and reduce the
right-to-left shunting of blood, improving oxygenation in a child with TOF during a cyanotic episode
27) Which of the following conditions would increase the risk of developing kernicterus in a jaundiced
neonate?
A) Maternal ingestion of phenobarbitone during pregnancy. (Sulfonamides can cause)
B) Neonatal Sepsis.
C) Metabolic Alkalosis.
D) Hyperalbuminaemia.
28) Which of the following statements is FALSE regarding childhood cancers in comparison to adult
cancers?
A) Overall incidence is much less in children compared to adults.
B) The cure rate of childhood cancer is comparatively better.
C) In both age groups, carcinomas are the predominant type.
D) Stage of the cancer is an important prognostic factor in both age groups.
30) Win, a 7 year old boy was diagnosed with tuberculosis and the doctor started anti-TB medications.
Two days later, he complained of orange discolouration of his urine. He is afebrile and physical
examination was normal. What is your next step of management?
A) Stop all anti-TB medication immediately.
B) Reduce the dose of all anti-TB medications.
C) Explain to the patient and parents that it is a side effect of Rifampicin.
D) Do Urinalysis to exclude Tuberculosis of the bladder.
DOC 19
A) Thalassemia
B) Chronic kidney disease
C) Vitamin B12 deficiency megaloblastic anemia → RBCs are larger and immature
D) Sideroblastic anaemia
2) A 5-day old newborn is brought with yellow discoloration of skin and sclera for the past 2 days. He
is feeding well and is active. He was born after 35 weeks of gestation with a birth weight of 1.8 kg. His
total serum bilirubin is 289mmol/dl (17mg/dl); Hb: 12gm/dl. The most appropriate mode of treatment
includes:
A) Blood transfusion
B) No intervention required
C) Phototherapy
D) Encourage frequent breast feeds important too, but since serum bilirubin is high phototherapy
is needed
3) A five-year-old girl was brought to the paediatric clinic for an acute episode of cough and
breathlessness. Which important point in the history would suggest that this is an acute asthmatic
attack?
4) A 3-year-old child was brought to the Child Health Clinic because her mother is worried that the
child is not growing as well as her two elder siblings. Her medical records were reviewed and
measurements done as given below. Her current weight for age is on the 3rd percentile.
Which of the child’s following features is suggestive that he is having failure to thrive?
A)Current head circumference at 5th percentile for age less than 5th percentile could be microcephaly
5) A 9 months old infant is brought with history of running nose and low grade fever for 3 days, and
abnormal inspiratory sounds heard by his mother since this morning. His body temperature is 38’C,
and his chest is clear on auscultation. His management should include:
A) Salbutamol nebulization
B) Cool mist inhalations
C) Antibiotics against group A β haemolytic streptococci
D) Single dose dexamethasone croup
6) Azliza is an infant who can sit forward on her hands, can reach for and grasp a cube and transfer it
from hand to hand and can babble. She can neither wave bye-bye, nor grasp objects between her
fingers and thumb. Her developmental age is most likely:
A) 7 month
B) 14 month
C) 10 month will be able to wave bye, pincer grasp
D) 5 month
7) 13-year-old child developed sudden onset of sore throat with bilateral swellings below and around
his ears. One week later his temperature was very high and he complained of severe epigastric pain
associated with bilateral scrotal swelling that was very painful and tender to touch.
A) Diabetes mellitus
B) Diphtiric pharyngitis
C) Mumps fever, sore throat, swelling of parotid glands. A complication seen → orchitis
D) (no ans given)
8) A 4-month-old infant is brought via ambulance to the ED intubated and with CPR in progress. The
parents report that he had been healthy and that they put him to bed as usual for the night at about
8pm. He did not awaken for his normal nightly feeding. When they next saw him in the morning, he
was not breathing. Physical examination in the ED shows only evidence of resuscitation including an
endotracheal tube from the mouth, an intraosseous line in the left leg. A film of the left leg from a
routine skeletal survey is shown in the photograph.
A) Congenital syphilis
B) Osteoporosis
C) Osteogenesis imperfecta
D) Non-accidental trauma shaken baby syndrome bc sudden rsp distress, resuscitation was
required and no hx of apparent medical cause or accidents
9) Arun is a nine-month-old infant who has been brought for vaccination to the clinic. The medical
officer examines him and finds his weight in the 50th percentile. He can sit unsupported and has a
good pincer grasp. He can speak in bisyllables, and his vital signs are normal. His chest is clear;
however, he has a loud pansystolic murmur in his left lower parasternal area with a thrill. The most
likely diagnosis is:
A) Increasing cyanosis
B) Bounding pulses
C) Increasing intensity of murmur
D) Hyperreflexia
A) Dengue fever
B) Mumps
C) Scarlet fever
D) Kawasaki disease
12) Elevated levels of cholesterol and triglyceride found in a 6-year-old boy whose mother reports that
he has been awakening with puffy eyes each morning. On physical examination, he has unexpected
weight gain.
13) 5 year old boy is admitted to the paediatric ward with a sudden onset of hematuria, edema and
hypertension. His mother claimed that he was having fever and sore throat 2 weeks ago and he had
received three days of ampicillin at the local clinic.
15) A 3 year old boy was admitted for the first time with pallor. There was no family history of anaemia
in the family. Features suggesting that he has haemolytic anaemia includes the following EXCEPT:
16) A 10-year old boy was brought to the hospital because of facial asymmetry which was confirmed
on careful cranial nerve examination. The diagnosis of right Bell’s palsy or idiopathic lower motor
neuron palsy the right facial nerve was made.
The evidence that it is the lower and not upper motor neuron lesion of the facial nerve is inability when
tested to
On examination he has audible wheeze, mild fever and respiratory distress. On auscultation of his
chest revealed that his breath sound was equal bilaterally and has prolonged expiratory phase. There
were bilateral rhonchi, but no crepitation was detected.
He would require
A) No further investigations
B) Chest radiography
C) Full blood count
D) Arterial blood gas analysis
18) A 3 year old boy was admitted with first episode of generalized seizures. The diagnosis of
meningitis is more like than that of febrile seizure if
19) A 2 year old Malay girl presented with pallor and mild hepatosplenomegaly. Her full blood count
showed the following results:
A) Machinery murmur
B) Is associated with left ventricular hypertrophy
C) Right to left shunt left to right
D) Common in Tetralogy of Fallot
DOC 20
1) An 8 year old presents with dark coloured urine and
mild swelling of her legs that she noticed 12 hours ago.
Her mother said that she had a sore throat 10 days ago
and the change in urine colour occured yesterday
evening. There is no history of trauma, flank pain,
frequency, urgency, dysuria or passing clots in the
urine. On examination, her blood pressure is 144/90
mm Hg and she has mild swelling of the face and lower
extremities.
Of the following, the MOST appropriate next step to
arrive at a diagnosis is?
A) Echocardiography.
B) Renal and Bladder Ultrasonography. Not
necessary
C) Throat swab for rapid streptococcal antigen.
Post-streptococcal acute GN so may not get
positive results
D) Estimation of serum creatinine levels.
Ans: Indications: prosthetic cardiac valves, pts with rheumatic heart disease, prev hx of infective
endocarditis, congenital heart diseases
Recommended antibiotics: oral/ IV amoxicillin or ampicillin 50 mg/kg
If allergic to ampicillin, oral/ IV clindamycin 20 mg/kg
5) A 2 week old neonate is brought by his mother to the clinic with complaints of constipation. She
says the child passes hard stools once in 2-3 days. He was born at 39 weeks via a normal vaginal
delivery with a birth weight of 3.2 kg and discharged the next day. On examination, he is a quiet child
who has jaundice till his thighs and his facial features look a little coarse. All the systems are normal
on examination. His weight is 4.0 kg (50th Centile). Your next course of action should be:
A) Ask her to get her son’s newborn screening test results.
B) Prescribe a laxative for constipation.
C) Reassure the mother and say it is normal.
D) Advise her to feed the neonate more frequently.
6) A 9 month old baby presented with a history of respiratory distress for the fourth time since birth.
His immunisation is up to date. He is mildly tachypnoeic and tachycardic with normal BP and oxygen
saturation. He is between the 3rd and 5th centile for weight. He takes 2 ounces of bottle feeds very
slowly, stopping multiple times and falling asleep before finishing it. There is a holosystolic blowing
murmur in the left lower parasternum. The most likely diagnosis is:
A) Ventricular Septal Defect.
B) Tricuspid Atresia.
C) Atrial Septal Defect.
D) Mitral Regurgitation.
7) A 9 year old girl was noted by her mother to have abnormal eye movement 3 weeks after
recovering from chicken pox. A diagnosis of post chickenpox encephalitis affecting mainly her
cerebellum is best suggested by the following findings:
A) Past pointing on the finger nose test.
B) Increased tone of her limbs.
C) Increased deep tendon reflexes.
D) Fine, static tremors of the hands.
8) A 12 month old boy has come to a well-baby check-up. The most appropriate milestone would be: -
A) He speaks three other words besides “Mama” & “Dada”.
B) He uses “Mama, Dada” constantly.
C) Mother says he has not started speaking anything.
D) He can speak 2-3 word phrases.
9) A first time mother has come with her one month old infant and is worried that her infant may not be
getting enough milk. The infant is exclusively breast fed. The baby is receiving enough milk if:
A) He gains 100 grams of weight every week.
B) He passes stools every alternate day. Usually passes stool everyday
C) He passes urine 6-8 times in 24 hours.
D) He sleeps for one hour after each feed.
10) A 2.5 month old male infant was admitted with respiratory distress and crackles all over the chest.
He is afebrile, pink in room air, and has a heart rate of 168 beats per minute. His apical impulse is
palpable in the 6th intercostal area lateral to the midline and liver is palpable 4 cm below the right
subcostal margin. Your management plan should be to:
A) Give nebulisation with bronchodilators.
B) Give supportive treatment for bronchiolitis.
C) Treat him with antibiotics for bronchopneumonia.
D) Give injection furosemide intravenously. Loop diuretic to prevent fluid overload
Ans: palpable in 6th intercostal space indicating cardiomegaly (normal in 4th intercostal space in
children below 3 yrs of age). Signs of heart failure in this pt: respiratory distress, crackles,
hepatomegaly
11) A 5 year old boy has been admitted with vomiting and diarrhoea for 2 days. He has vomited 5
times and has passed watery stools 7-8 times in the past 2 days. He has passed dark coloured urine
once this morning.
On examination, the temperature is 38.2°C, BP is 90/65 mm Hg. He weighs 18 kg (50th percentile).
He is alert and cooperative but looks tired. His oral mucosa is moist but he has just vomited. His neck
is supple. Heart and lung examinations are normal. His abdomen is soft and non-tender. His capillary
refill time is 2 seconds over his chest and his hands and feet are cold.
His management should include:
A) Intravenous Ringer Lactate 360 ml over 30 minutes.
B) Intravenous 0.9% saline 360 ml over 30 minutes.
C) Oral rehydration solution 50 ml/kg over 4-5 hours. Moderate dehydration
D) Oral rehydration solution 10 ml/kg over 4-5 hours.
12) An 11 month old boy weighs 7.5 kg and has iron deficiency anemia with a Haemoglobin of 4.2
g/dL. His heart rate is 188/minute. You decide to transfuse him. Which is the best way to transfuse
him with packed red blood cells?
A) Transfuse 20 ml/kg over 12 hours with furosemide injection.
B) Transfuse 15 ml/kg slowly, then start oral iron.
C) Transfuse 5 ml/kg over 4 hours, may repeat the next day.
D) Transfuse 10 ml/kg over 4 hours with furosemide injection.
13) A 11 month old infant is brought to the emergency room with a large bruise on his thigh without
any obvious history of trauma. Coagulation tests reveal that the factor VIII level is less than 1%
(normal 70-130%). You are required to counsel the parents on the risk of haemophilia in their yet
unborn child. The risk of haemophilia in the yet unborn baby is:
A) 25% regardless of the sex of the offspring. X-linked recessive disorder so affects males more
B) 25% for a female offspring, 50% for a male offspring.
C) No risk if the father is genetically normal. If mother is a carrier, still a 50% chance of unborn
sun may have hemophilia, 50% chance of daughter being carrier
D) None for a female offspring, 50% for a male offspring.
Ans: An affected boy in the first pregnancy, so mother is a carrier
14) A 4 year old boy is brought to the emergency department because of fever, malaise and
photophobia of 4 days duration. This morning, his parents noticed a rash on the face and neck. The
child has not received all the necessary immunisations. His temperature is 40.1°C. He has significant
conjunctival injection bilaterally. There is a maculopapular erythematous rash on the forehead and
behind the ears, descending on the sides of the neck. Which of the following vaccines would have
prevented this disease?
A) Haemophilus influenzae Type B vaccine.
B) Varicella vaccine.
C) Meningococcal vaccine.
D) Measles-Mumps-Rubella vaccine.
15) The parents of a 2 year old boy have come to see their family doctor about their son whose
recurrent episodes of wheeze is causing concern. The first episode of wheeze occurred at the age of
6 months. This was diagnosed as bronchiolitis. In addition to several admissions for “viral associated
wheeze” and “asthma”, there are several primary care consultations for “wheeze” and “chest
infections”. These have been variously treated with bronchodilators, steroids (inhaled and oral) and /
or antibiotics - None of which have had any convincing effect. He has no family history of atopy.
On examination, the child’s temperature is 38°C, respiratory rate is 45/min, heart rate is 110/minute.
He is on the 25th centile of his weight curve. Cardiac examination shows his apex beat is in the 4th
intercostal space on the mid-clavicular line. Auscultation of the chest reveals generalised ronchi and
occasional crackles. Doubt
What is the best management strategy for this boy?
A) Supportive treatment when the attack occurs.
B) Short acting inhaled steroids as a long-term reliever.
C) A course of antibiotics for 2 weeks.
D) Long acting β-2 agonist when attacks occur.
16) A previously well 5 month old girl is referred by her general practitioner with pyrexia. Her parents
report that she has been having spiking temperatures for the past 2 days and is feeding less than
usual. She has been vomiting, although there is no history of diarrhoea. She has a temperature of
38.8°C and her vitals are normal. She is sleeping but easily arousable. She has a normal cry, normal
movements and a soft fontanelle. She has a full, wet nappy. Systemic examination does not reveal
any abnormalities. The diagnostic test to aid the diagnosis should be:
A) X-Ray Chest supine film.
B) Urine for microscopy and culture. Suspected UTI
C) Nasopharygeal swab aspirate for viral PCR. no symptoms like runny nose/ cough
D) Cerebral-spinal fluid examination.
17) A 5 year old girl was referred to your hospital with the symptoms of fever and vomiting for 48
hours and altered sensorium for the last 12 hours. Her growth and development were appropriate for
her age. Her heart rate was 128/min, respiratory rate was 41/min, blood pressure was 97/38 mm Hg.
She had sunken eyes, dry tongue and a wound over the left knee. Her lungs were clear, and
abdomen was distended. She was drowsy and had generalised hypotonia. Pupils were bilaterally
equal and reactive to light. Her fundus examination was normal. The most important investigation to
arrive at a diagnosis is:
A) Urine culture and sensitivity.
B) Blood culture and sensitivity.
C) Cerebral-spinal fluid examination.
D) CT Brain.
20) A 6 week old baby was brought for vaccination to the health-clinic (klinik kesihatan). On
examination, he had a large tongue, umbilical protrusion, a large anterior fontanelle and jaundice. On
questioning, his mother said he passed stools once in 3-4 days. The baby should be tested for:
A. Serum calcium.
B. Thyroid function. Signs of congenital hypothyroidism
C. Glucose-6-phosphate dehydrogenase.
D. Serum iron and ferritin.
DOC 21
1) A 1-year-old infant weighing 11kg has been admitted with AGE. Since his diapers were
soaked with stools, his mother does not know when he passed urine last. On examination he
is drowsy but responsive, his fontanel is sunken, and peripheries are cold. His capillary refill
time is > 5 seconds. His heart rate is 166 bpm, respiratory rate 54/min, and he is afebrile. He
should receive immediately 220ml of:
a. 0.9% saline over one hour
b. 10% dextrose over one hour
c. 0.45% saline with 5% dextrose over one hour
d. Harman’s solution over one hour
2) The infant in the case above has now been rehydrated over the last 5 hours, but he has not
passed urine. His vitals are now stable and he is alert. You have passed a urinary catheter but his
bladder is empty. The next step in management should be to:
A. Stop the IV fluid and watch for urine output for an hour
B. Increase the amount of IV fluids to 1.5 times of maintenance
C. Start oral rehydration solution along with maintenance IV fluid
D. Give injection furosemide IV stat
3) A 2-month-old baby is brought by her 18-year-old single mother early in the morning saying she
has accidentally scalded her baby’s right hand and arm while she was pouring hot coffee and
soothing a crying baby at the same time. You will suspect child abuse if the burn
5. Shivani is a 10-year-old girl whose teacher has complained that she is sleepy in the
classroom and does not pay attention. Her parents noticed her grades decreasing for the past
one year. On questioning they said she snores during sleep and is very restless in her sleep.
She has been obese since early childhood. Her present weight is 48kg (>95th percentile) and
height is 152 cm (>95th percentile). There has been no significant past history and no hospital
admission before. Systemic examination reveals no abnormalities. Further course of action
for her management should be:
a. Advise on lifestyle modification
b. Investigate her for Cushing syndrome
c. Advise her to use CPAP at night try lifestyle modification first
d. Exclude T1DM
6. An infant can babble and has an ulnar grasp. About 2 weeks ago has started transferring
objects from one hand to the other. He should be also be able to
a. Build a tower of 2 blocks
b. Sit unsupported
c. Walk with support
d. Say “mama”, “dada” and “doggie”
7. All these chromosomal defects can be seen in children with Down syndrome except
a. Trisomy 21
b. Mosaicism
c. Robertsonian translocation
d. Fragile X
8. A 15-month-old baby boy was brought to klinik kesihatan because of profuse watery
diarrhoea for 2 days. There was no blood or mucus in the stools which were yellow in colour.
The baby was very thirsty and drank vigorously and subsequently vomited. His weight was
9.3kg (it was 10kg 3 days prior to diarrhoea). The fontanel was sunken and oral mucosa was
dry. Skin turgor was less than 2 seconds. He had passed dark-coloured urine about 4 hours
ago. State the hydration status in the baby:
A. No dehydration
B. Severe dehydration
C. Moderate dehydration
D. Mild dehydration
A. Iron deficiency
B. Thalassemia
C. Lead poisoning
D. Chronic renal failure
Doc 22
1. Developmental delay
A. Abnormal head size B. primitive reflex persistent more than 6 months
C. Not walking by 11 months D. No clear spoken words by 8 months
2. 5 y/o child presented with jaundice, pallor and dark coloured urine. All other systems are normal
A. Glutathione deficiency B. G6PD deficiency
C. Haemoglobin H D. Hemolytic uremic syndrome
7. 8 y/o child presented with breathlessness and headache. He has jaundice for 3 days. BP 140/90.
Respiratory rate is 36. Pulse is 140. What would you look for during examination? - not sure
A. splenomegaly B. Bilateral Bibasal Crep C. delayed CRT
8. Which of the following is the cerebellar signs :
A. intention tremor B. Chorea
9. Contraindication of ORS:
A. Shock
10. 7 month old previously well child, has high grade fever for 3 days 39.4C, vomiting ( forgot how
many time ) and difficulty in sucking. On examination, anterior fontanelle soft, pulse rate is 140 bpm,
BP 147/95. The most possible diagnosis - not sure
A. neonatal hepatitis B. UTI
C. AGE D. Acute meningitis
15. true
A. pancarditis – rheumatic B. exudative pharyngitis- Kawasaki
C. isolated RVH – PDA
SBA Y5R2
1. Chicken pox.
A.
B. He is not infective 5days after the appearance of the first lesion.
C. He is no longer infective once there is no fresh vesicles.
D. He is no longer infective once all the lesions have crusted and dried. (ans)
3. A previously well 5m/o girl has fever x2/7. She is vomiting, but has no diarrhea. She is sleeping but
easily aroussable. She has a normal cry, normal movements and a soft fontanelle. Systemic
examination does not reveal any abnormalities. The most appropriate Invx:-
A. CXR B. LP C. Blood C&S D.
4. Serum ferritin 102ng.ml (30-335 normal), iron 185ug/dl (50-150 normal), total Hb 7.8g/dl, total
bilirubin 5.3mg/dl. Diagnosis
A. Thalassemia intermedia (ans) B. Sickle cell anemia
C. Barts hemoglobinopathy D. Hb E disease
5. A 10y/o girl admitted with H/O fever and SOB x3/52. She was OP for complex CHD at 18m/o and
again at 7y/o. O/E, she is pale and 38 temp., PR 126bpm, RR 22, BP 104/60mmHg. On ausc., there is
a grade3 ESM best heard at left 3/4th ics, spleen is palpale 3cm. She has microscopic hematuria. Diag
A. IE (ans) B. pyelonephritis
C.septicemia D. Kawasaki disease
6. A 16m/o orang asli (aboriginal) child has a hx of passing 10-15 loose stools daily x2/7. He also has
5-6 epi vomiting on the 1st day of illness. O/E, h is afebrile, with cold extremeties, and CRT of 6sec.
His mother says she cannot remember when he passed urine last. He has fast breathing. His blood
invx is likely to reveal: -
A. hypocalcemia B. hyperglycemia
C. hypophosphatemia D. metabolic acidosis (ans)
8. ?? genetic disease
A. mother is carrier, 50% of offspring will affected. B. mother is carrier, all daughters are carrier.
C. father is affected, all daughters are carriers. (ans) D. mother is carrier, all sons are affected.
10. A 3m/o child admitted for 2nd time with resp distress. His birth wt is 3.56kg. O/E, he is afebrile, RR
64, HR 172bpm, and wt is 2.8kg. He has signs of resp distress. His apical impulse is in 6th ics lateral to
mcl. Ausc, lungs have crackles all over, liver was palpable 4cm below costal margin. Diagnosis.
A. Bronchopneumonia B. CHD with cardiac failure (ans)
C. acute bronchiolitis D. asthma
12. A 8w/o female child admitted with resp distress x3/7. She was delivered at term, birth wt of
3.2kg and was exclusively breast fed. Her Hb 9.3g/dl; total WC 9500/mm3; differential count N 82%, L
18%. CXR reveals bronchopneumonia, she is started on antibiotics. In view of her Hb levels: -
A. she should be gien a packed RBC transfusion. B. She should be prescribed iron drops.
C. she should be invx for occult blood loss. D. She needs no intervention. (ans)
13. A 15y/o girl has a 2/52 hx of fatigue & back pain. She has bruising, pallor and tenderness over her
vertebrae and both femurs. Her liver is palpable 4cm and spleen 6cm below costal margin. Hb 5g/dl,
WCC 2000/mm3, plt 15000/mm3. She should be invx for: -
A. infectious mononucleosis B acute leukemia (ans)
C. meningococcemia D. ITP
15. ??
A. start him on full dose prednisolone. B. give him another course of cyclophosphamide.
B. assess his renal function. D. perform a renal biopsy.
Doc 23
Q1. An 8-year-old girl presents with dark-colored urine and mild swelling of her legs that she
noticed 12 hours ago. Her mother said that she had a sore throat 10 days ago and the
change in urine color occurred yesterday evening. There is no history of trauma, flank pain,
frequency, urgency, dysuria or passing clots in the urine. On examination, her blood pressure
is 144/90 mmHg and she has a mild swelling of the face and lower extremities. Of the
following, the MOST appropriate next step to arrive at a diagnosis is:
Q2. A 22-year-old lady has delivered her first healthy male newborn just yesterday and
wants advice on breastfeeding. Your advice should include:
Q3. A 2-week-old neonate is brought by his mother to the clinic with a complaint of
constipation. She says the child passes hard stools once in 2-3 days. He was born at 39
weeks of a normal vaginal delivery with a birth weight of 3.2 Kg and discharged the next day.
On examination he is a quiet child who has jaundice till his thighs and his facial features look
a little coarse. All the systems are normal on examination. His weight is 4.0 Kg (50th centile).
Your next course of action should be:
A. Reassure the mother and say it is normal
B. Prescribe a laxative for constipation
C. Advise her to feed the neonate more frequently
D. Ask her to get her son’s newborn screening test results
Q4. A 9-month-old baby presented with a history of respiratory distress for the fourth time
since birth. His immunization is up to date. He is mildly tachypneic and tachycardic with
normal BP and oxygen saturation. He is between 3rd and 5th centile for weight. He takes 2
ounces of bottle feeds very slowly, stopping sometimes and falling asleep before finishing it.
There is holosystolic blowing murmur in the left lower parasternal area. The most likely
diagnosis is:
A. Hypertension
B. Bilateral cataract
C. Osteoarthritis
D. Gallstones
Q7. A 12-month boy came for a well-baby checkup. The most appropriate milestone would
be:
Q8. A 2.5 month-old male infant was admitted with respiratory distress and crackles all over
the chest. He is afebrile, pink in room air and has a heart rate of 168 beats/minute. His apical
impulse is palpable in the left 6th intercostal area lateral to the midline and liver is palpable 4
cm below the right subcostal margin. Your management plan would be:
Q10. A first-time mother has come with her one-month-old infant and is worried that her
infant may not be getting enough milk. The infant is exclusively breast fed. The baby is
receiving enough milk if:
Q11. A 11-month-old infant is brought to the emergency with a large bruise on his thigh
without any obvious history of trauma. Coagulation tests reveal that the factor VIII level is
less than 1% (normal 70 to 130%). You are required to counsel the patients on the risk of
hemophilia in their yet unborn child. The risk of hemophilia in the yet unborn baby is:
Q12. A 4-year-old boy is brought to the emergency department because of fever, malaise
and photophobia. This morning his parents noticed a rash on the face and neck. The child
had received all the necessary immunizations. His temperature is 40.1°C. He has significant
conjunctival injection bilaterally. There is a maculopapular erythematous rash on the
forehead and behind the ears descending on the sides of the neck. Which of the following
would have prevented this disease?
A. Measle-mumps-rubella
B. Meningococcal vaccine
C. Varicella vaccine
D. HIB vaccine
Q13. Hepatitis B viral infection:
Q14. A 5-year-old boy has been admitted with vomiting and diarrhea for two days. He has
vomited 5 times and has passed watery stools 7-8 times in the past 2 days. He passed
dark-coloured urine once this morning.
O/E: T: 38.2°C, BP: 90/65 mmHg. Weight is 18 Kg (50th percentile). He is alert and
cooperative but looks tired. His oral mucosa is moist, but he has just vomited. His neck is
supple (not stiff). Heart and lung examinations are normal. His abdomen is soft and
nontender. His capillary refill time is 2 seconds over his chest and his hands and feet are
cold. His management should include:
Q15. An 11-month-old boy weighs 7.5 Kg and has iron deficiency anemia with a Hb of 4.2
g/dL. His HR is 168 bpm. You decide to transfuse him. Which is the best way to transfuse
him with packed red blood cells.
Book says less than 5 ml/kg over 4-6 hours with IV Furosemide 1 mg/kg midway
Q16. A 5-year-old girl was referred to your hospital with the symptoms of fever and vomiting
for 48 hours and altered sensorium for the last 12 hours. Her growth and the development
were appropriate for her age. Her HR was 126 bpm, RR 41/min, BP: 97/38 mmHg. She had
sunken eyes, dry tongue and a wound over the left knee. Lungs clear, abdomen distended.
She was drowsy and had generalized hypotonia. Pupils were bilaterally equal and reacting
to light. Her fundus examination was normal. Most important investigation to arrive at the
diagnosis:
A. Urine culture and sensitivity
B. CSF examination
C. Blood culture and sensitivity
D. CT brain
B.
Q17. Acute fulminant hepatitis is characterized by:
A. Splenomegaly
B. Anemia
C. Encephalopathy
D. Hepatomegaly
C.
Q19. The parents of a 2-year-old boy have come to see their family doctor about their son
whose recurrent episodes of wheeze are causing concern. The first episode of wheeze
occurred at the age of 6 months. This was diagnosed as bronchiolitis. In addition to several
admissions for ‘viral associated wheeze’ and ‘asthma’, there are several primary care
consultations for ‘wheeze’ or ‘chest infections’. These have been variously treated with
bronchodilators, steroids (inhaled and oral) and/or antibiotics - none of which have had any
convincing effect. He has no family history of atopy.
Q20. A previously well 5-month-old girl is referred by her general practitioner with pyrexia.
Her parents report that she has been having spiking temperatures for the past 2 days and is
feeding less than usual. She has been vomiting, although there is no history of diarrhea. She
has a temperature of 38.8°C and her vitals are normal. She is sleeping but easily arousable.
She has a normal cry, normal movements and a soft fontanel. She has a full wet nappy.
Systemic examination did not reveal any abnormalities. The diagnostic test to aid the
diagnosis should be:
B
Doc 24
Q1. A 5-month-old female is admitted for the third time with a history of respiratory distress
for the past 2 days. She is the 3rd child of a 29-year-old mother, who says she takes a long
time to feed, and the mother has noticed beads of sweat on her forehead while feeding.
On examination she is alert and warm to touch. Her HR is 168 bpm, RR 70/min and her
weight is below the 3rd percentile. Her apical impulse is in the 7th intercostal space in the
anterior axillary line. Auscultation of the lungs reveals widespread crackles and rhonchi. The
first step in management should be:
C.
Q2. A three-month-old girl infant has been brought for vaccination to the well-baby clinic.
She is active and growing well, but the medical officer thinks she looks pale, so he gets her
blood counts done. Her total white cell count and differential count were normal, but her
hemoglobin was 9 g/dL. All her systems are normal on examination. Her management
should include:
D. Other than pallor, she has no other signs or symptoms of anemia. For 3-month-olds Hb
should be between 9.5 - 14.5, so it is not that low. And this is only one occasion.
Q3. A 13-year-old boy presents to his primary care provider with a 3-day history of
abdominal pain and diarrhea. He has a dull ache in the epigastric region, not related to
movement. His mother discloses that several family members at the time also have episodes
of diarrhea. On examination, he is alert and oriented but looks tired. He is afebrile, all vital
signs are stable and his lips and mouth are dry. His neck is supple. Heart and lungs are
clear. His abdomen is flat, soft and nontender. The management should include:
Q5. Developmental assessment of a three-year-old girl reveals that she can ride a tricycle
and dress herself. She can speak words like “Mama”, “Dada” and “Doggie”. You will classify
her developmental as:
C. Speech delay. By three years, speech should be half-understandable and the child should
be able to name 4 objects.
A. Sterile pyuria
B. Hemorrhagic cystitis due to the adenovirus
C. Acute post-infectious GN
D. Lower UTI with no anatomical abnormality.
C.
Q7. A 12-year-old boy came to the ED with a history of recurrent swelling of the left knee
joint for the last 7 years. Each episode of swelling improved after he was given Factor VIII
infusion. His maternal grandfather has the same symptom, but the swelling often occurred in
his right elbow. State the type of inheritance of the disease in the patient.
A. Autosomal recessive
B. Autosomal dominant
C. X-linked recessive
D. DNA linked recessive
C.
Q8. An 8-year-old boy came to KK with the following hepatitis B serology results.
Positive HBsAg
Positive IgM and HBc and
Negative anti-HBs
Antigen positive, IgM positive: current infection, negative anti-HBs means that it has not
reached the convalescent stage yet. Anti-HBc means have been infected with Hep B.
Q9. Jamil is a 5-year-old boy brought to the clinic by his mother. She is concerned since she
has noticed that he is becoming increasingly tired, and no longer enjoys playing outside. His
kindergarten teacher has noticed the same. O/E: pallor, Temp: 38°C. He has some bruises
on his lower limbs and visceromegaly. The rest of the examination is unremarkable. As a
primary care physician what should be your first step?
A. Send an FBC. I think I'm suspecting ALL because of pallor, reduced activity, and
visceromegaly and bruises.
Q10. A six-month-old male presents to the ED with a history of lethargy. He was seen 3 days
ago with fever and URTI symptoms, diagnosed with otitis media, and treated with oral
amoxicillin. This morning he had become irritable and was less active than usual. He has
vomited thrice and his urine output is noticeably decreased. He has no diarrhea. What
additional finding will make you suspect meningitis?
A.
C.
Q14. A 15-month-old male infant was admitted with respiratory distress and crackles all over
the chest. He is afebrile, pink in room air, and has a HR of 166 bpm, RR 70/min. His apical
impulse is palpable in the left 6th ics lateral to the midline and liver is palpable 4 cm below
the right subcostal margin. You management plan should be to:
O/E, RR 50/min and is using his accessory muscles of respiration and has nasal flaring. He
has an audible wheeze. He can talk but only in single words. His oxygen saturation is 88%
and HR is 160 bpm. On arrival to the emergency he is started on high flow oxygen and has
already been given nebulized salbutamol once. His respiratory rate after 30 minutes is
48/min and rhonchi are heard all over his chest.
The next step in management should be:
Q16. Despite treatment in the previous question, he fails to improve and his SPO2
decreases to 85%. The next step in management should be:
Q17. The parents of a 3-year-old boy have come to you about their son whose recurrent
episodes of wheezes and respiratory symptoms are causing concern. The first episode of
wheeze occurred at the age of 6 months and several subsequent visits to the ED or clinics
have been treated with bronchodilators, steroids (inhaled and oral) and/or antibiotics - none
of which have had any convincing effect.
A. Viral-induced wheeze
B. Bronchial asthma
C. Bronchopneumonia
D. Foreign body aspiration
Q18. A 2-year-old patient with acyanotic congenital heart disease has a continuous murmur
in the upper left sternal edge. His apex beat and heart sounds are normal. There is no other
murmur heard. The following finding on peripheral pulse examination will help to diagnose
the cause of the murmur:
A. Collapsing pulse
B. Radial femoral delay
C. Small volume pulse
D. Irregular pulse
Q19. A 9-year-old girl was noted by her mother to have abnormal eye movements about
three weeks after having chicken pox. A diagnosis of post chickenpox encephalitis affecting
mainly her cerebellum is best suggested by the following EXCEPT
Q20. A 12-year-old boy presented with acute weakness of both his lower limbs. O/E the
etiology was suspected to be due to transverse myelitis i.e. spinal cord pathology, at the
thoracic spine. The spinal level or site of the lesion is best assessed by examining the
Doc 25
Q1. A 6-year-old girl has diarrhea for 2 days. O/E she is unconscious, lips and tongue are
dry, there is loss of skin turgor. Blood pressure is 70/50 mmHg, pulse is weak and CRT >2 s.
The extremities are cold and body temperature is 37.5°C. She has not passed urine for the
past 5 hours. The etiology of the anuria is:
D. Pre renal: severe dehydration -> hypovolemic shock -> decreased renal perfusion
Q2. A two-year-old had loose stools for the past three days. She is the fourth child of a
rubber tapper father and a home-maker mother. Her weight is 7 Kg (below 3rd percentile)
and height is 78 cm (below 3rd percentile). She is afebrile, but lethargic. Her pulse is 126
bpm and of low volume. RR 40/min, hands and feet are cold with a CRT of 4-5 seconds. She
is pale. The goals for her immediate management should be:
Q3. A 7-year-old boy comes for a preschool enrollment physical examination. During routine
questioning his mother says he is not interested in sports or physical activities and prefers
reading books. There is no past history of sore throat or joint pains. O/E: he is afebrile, HR
96 bpm with normal volume pulse. RR is 22/min, lungs are clear. Height and weight are at
the 25th percentile. ENT examination is normal. Examination of the CVS reveals a normal
sized heart with normal S1 and S2. A grade 3/6 ejection systolic murmur is heard best at the
left upper parasternal area. ABD examination is normal. The likely diagnosis is:
A. ASD
B. PDA
C. ToF
D. Idiopathic pulmonary arterial hypertension
Q4. Shivani is a 10-year-old girl whose teacher complains that she is sleepy in class and
does not pay attention. Her grades have been decreasing over the past one year. She
snores and is restless in her sleep. She has been obese since early childhood. Her present
weight is 48 kg (>95th percentile). There is no significant past-history or hospital admission.
Systemic examination reveals no abnormalities. The appropriate counseling for her is:
A. Tonsillectomy
B. Use of CPAP at night
C. Investigation for Cushing syndrome
D. Reduction in weight through diet modification
A. Presence of anemia
B. Peak serum bilirubin levels at day 10 of life
C. Jaundice appearing on 2nd-3rd day of life
D. Lethargy in the baby
Q6. A 6-year-old boy with acute lymphoblastic leukemia in the induction phase has fever,
cough and runny nose since this morning. His brother has fever and runny nose as well. On
examination, body temperature is 38°C. Child is alert. There is no nasal flaring. On
auscultation, there is prolonged expiration. Hb is 110 g/L, WBC 2 x 10^3 / uL, platelet 150 x
10^3 / uL. The appropriate immediate management is:
A. Start IV antibiotic
B. Reassure the parent it is nothing serious
C. Do a bone marrow examination
D. Antipyretic and plenty of fluids
A. IV antibiotic, cause the child has neutropenia, very scary if fever, complications.
Q7. Amir, 1-month old baby boy was taken for immunization. According to the mother, the
baby feeds well on the breast and is active. One examination, the infant was pink, his pulse
rate was 136 beats per minute, regular and the respiratory rate was 44 breaths per minute.
On auscultation, there is a machinery murmur, grade 3/6 in the left upper parasternal area.
State the diagnosis of the baby:
A. ToF
B. PDA
C. TGA
D. VSD
Q8. Name one of the contraindications for breastfeeding
Q9. A 16-month-old child is brought from a village with a history of passing 10-15 loose
stools daily for the last two days. He also had 5-6 vomiting on the first day of illness. On
examination he is afebrile, drowsy with cold extremities and has a capillary refill time of 6
seconds. His mother says she cannot remember when he passed urine last. His blood
investigations are likely to reveal:
A. Hypophosphatemia
B. Hypercalcemia
C. Metabolic acidosis
D. Hyperglycemia
C. Metabolic acidosis
Q10. A 15-month-old baby boy was brought to the KK because of profuse watery diarrhea
for 2 days. There was no blood or mucus in the stools which were yellow in color. The baby
was very thirsty and drank vigorously and subsequently vomited. His weight was 9.3 kg (it
was 10 kg 3 days prior to diarrhea). The fontanel was sunken and the oral mucosa was dry.
Skin turgor was less than 2 seconds. He had passed urine less than 3 hours ago.
State the hydration status of the baby:
A. No dehydration
B. Severe dehydration
C. Mild dehydration
D. Moderate dehydration
Doc 26
1. The infant in the previous case (what case) has now been rehydrated over the last 5
hours. But he has not passed urine. His vitals are now stable and he is alert. You
have passed a urinary catheter but his bladder is empty. The next step in
management should be to:
A. Increase the amount of IV fluids to 1.5 times of maintenance
B. Stop the IV fluids and watch for urine output
C. Start oral rehydration solution along with IV fluids
D. Give injection furosemide IV once.
D. Lms answer.
3. Elevated levels of cholesterol and triglycerides found in a 6-year-old boy whose mother
reports that he has been awakening with puffy eyes each morning. On examination, he has
unexpected weight gain. Which of the following is the most likely next step?
A. Urine culture
B. Echocardiography
C. Renal biopsy
D. Urine analysis
D. Urine analysis to see if he has proteinuria.
4. A 7-year-old girl presented with headache, decreased urine output and dark colored urine.
The most likely finding on your physical examination would be
A. Sacral edema
B. Periorbital edema
C. Ankle edema
D. Pulmonary edema
B. Periorbital, scrotal/labial. Scenario supports more towards nephritic syndrome due to
oliguria, hematuria and also symptom of hypertension which is headache.
Q1. A 3-year-old boy has come to the emergency department with a history of fever and
generalized tonic clonic seizure lasting for 5 minutes. Post ictally, the patient regains
consciousness and plays as usual. His father had similar symptoms while he was at his age.
On examination he has no nuchal rigidity. Brudzinski and Babinski signs are absent. Name
the next step to do.
A. Reassurance to the parent
B. CT scan
C. EEG
D. Lumbar puncture
a. Not sure. This patient most likely had a simple febrile seizure. No reason to suspect
meningitis.
Q2. A 5-year-old boy has come to the pediatric clinic with a 2-week history of excessive
eating and drinking. Having been dry at night for some time, he has also started wetting the
bed. His mother thinks that he has lost some weight. He has been less cheerful than usual.
On examination he is playing happily in the playroom. His weight is in the 10th centile and
his height is in the 50th centile. He is afebrile and not dehydrated. His pulse is 84 beats/min
and his capillary refill time is less than 2 seconds. Examination of the respiratory and
abdominal systems is normal. What is the next laboratory investigation he should take?
A. Full blood count
B. Urine analysis
C. Serum electrolyte
D. Blood glucose
D. Blood glucose. Seems like a presentation of DM type 1 in a child. Polyphagia, polyuria,
weight loss.
Q3. A 5-month-old baby boy was brought to KK by his mother with pallor which she noticed
since he was two months old. The baby was exclusively breast fed since birth. On
examination the baby looked pale and had a grade 2/6 murmur in the left parasternal area.
Liver was palpable 3.5 cm below the right costal margin and the spleen was just palpable.
Blood count showed Hb 7 g/dL, total white cell count 9.0 x 10^3/uL and platelet count 170 x
10^3/uL. Blood film showed anisocytosis, poikilocytosis, and microcytic hypochromic red
blood cells. No blast cell was seen. Name the investigation that should be done to confirm
the diagnosis.
A. Serum ferritin
B. Serum iron
C. Total binding iron capacity
D. Hemoglobin electrophoresis
D. Thalassemia confirmatory test. IDA or Thalassemia. But IDA normally after cow milk is
introduced, after 6 months.
Q4. A 5-year-old girl came to the pediatric clinic with high fever for 6 days. On examination
she had left cervical lymphadenopathy that was non-tender, mobile, measuring 6 cm and
non-purulent hyperemic conjunctiva. She also had a red inflamed tongue, pharynx and
tonsils. Her hands were swollen. State the most likely diagnosis of the patient.
A. Acute viral conjunctivitis
B. Scarlet fever
C. Measles
D. Kawasaki disease
D. Kawasaki. Fever for more than 5 days with cervical lymphadenopathy more than 1.5 cm,
conjunctival injection, red inflamed oral mucosa/lips, extremities swollen.
Q5. An 8-year-old boy came to the KK with the following hepatitis B serology results. HBsAg
+ve, IgM anti-HBc +ve, negative anti-HBs. Interpret the serology result.
A. He has acute hepatitis B infection
B. He is immune to Hepatitis B due to natural infection
C. He has chronic hepatitis B infection
D. He is immune to Hepatitis B due to vaccination
A. Acute hepatitis B. IgM positive, but no anti HBs. And HBsAg is positive.
Q6. A 15-year-old girl has a two-week history of fatigue and back pain. She has widespread
bruising, pallor and tenderness over the vertebrae and both femurs. She has a liver palpable
4 cm below the costal margin and a hard spleen measuring 6 cm. Complete blood count
shows hemoglobin concentration of 50 g/L, leukocyte count of 2 x 10^9/L and platelet count
of 15 x 10^9/L. She should be investigated for
A. Meningococcemia
B. Infection mononucleosis
C. ITP
D. Acute leukemia
D. Acute leukemia. Hepatosplenomegaly and low platelet and Hb and WBC.
Doc 29
1.A 7 year: old boy presents to the clinic with a history of passing dark coloured urine
for 3 days, and having headache and vomiting since this morning. His heart rate is
60 bpm and BP is 140/92 mmHg. Management strategy should include:
A Prednisolone
B Enalapril
C Gentamicin
D Diclofenac Sodium
Ans: B
2. A normal infant can sit forward on her hands, can reach for and grasp a cube and
transfer it from hand to hand, can babble but not wave bye-bye, nor can she grasp
objects between her fingers and thumb. Her age is:
A 4 months
B 7 months
C 10 months
D 14 months
Ans: B
Pg 42 sunflower
3. A 15 year old girl has two week of history of fatigue and back pain. She has
widespread bruising, pallor and tenderness over the vertebrae and both femurs. She
has a liver palpable 4cms below the costal margin and a hard spleen measuring 6
cms. Complete blood count shows haemoglobin concentration of 5.0g/dL. Leukocyte
count of 2000/mm3 and platelet count of 15,000/mm3. she should be investigated for:
A Infectious mononucleosis
B Acute leukemia
C Meningococcemia
D Idiopathic thrombocytopenic Purpura
Ans: B
4. A 2 year old boy on a normal diet is an active child. At the age of 1 year his Hb
10.2gm/dl and he was put on iron drops by his pediatrician. 3 months later his Hb
was still 10.3 gm/dl. His mother continued his iron drops until now. His current blood
tests now showed:
Haemoglobin 10.3gm/dL
Haematocrit 31%
Mean corpuscular volume 58fL
Reticulocytes 6%
Total leucocyte count 10.5 x 109/L
Differential count normal
The investigation most likely to reveal the cause of his anemia is:
5. A 5 days old male neonate presented with jaundice for 2 days. He was born at 35
weeks gestation with a birth weight of 1.8kg. His total serum bilirubin is 17mg/dl
(289mmol/dl) and Hb is 11.5gm/gl. He is active and feeding well. Choose the most
appropriate mode of treatment.
A No intervention required
B Phototherapy
C Blood transfusion
D Exchange transfusion
Ans: B
6. An uncomplicated VSD in a 6 year old boy child may be associated with:
A A collapsing pulse
B Wide and fixed splitting of the second heart
sound
C Clubbing of the fingers
D A pansystolic murmur in the mitral area
Ans: D
7. A one month old infant is brought to the clinic for fever of 3 days duration. He was
born at term via SVD after a normal pregnancy and is on formula feeds. Several of
his family members have URTI for the past week. On examination he is awake and
alert and is feeding well. His temperature is 38.90C, RR is 58/min, and all the
systems are within normal limits. Most appropriate management will be:
8. A 7 year old boy is admitted with a history of high fever for the past 5 days and
extreme lethargy since this morning. Dengue infection is suspected if you find:
A Anemia
B Splenomegaly
C Hypertension
D hepatomegaly
Ans: D
9. A previous healthy 10 month old infant is brought by his mother with history of low
grade fever and cough and running nose for the past 3 days, and rapid breathing
since last night. On auscultation high pitched rhonchi are heard all over the chest.
The most likely diagnosis is:
10. A 6 year old girl is referred to the clinic with complaint of generalized weakness,
vomiting and polyuria. She is on the 10th percentile of growth chart. She is alert and
lethargic with moderate dehydration and no systemic findings. Her PR is 130bpm,
RR is 38 breaths/mins, and BP is 90/74. The investigation to be requested to arrive
at a diagnosis is:
11. A 3 months old infant is brought to A&E with tonic clonic seizure lasting for 2
minutes. He is having diarrhoea for the past 3 days and he has been fed only water
& orange juice. The most likely cause of his seizure is:
A Hyponatremia
B Hypernatremia
C Hyperkalemia
D Hypokalemia
Ans: A
12. A 10 year old girl is admitted with history of low grade intermittent fever and
breathlessness for the past 3 weeks. She was operated for complex CHD at the age
of 18 months and again at 7 years. On examination she is pale and has temperature
of 38oC. Her PR is 126bpm, RR is 22/min, and BP is 140/60mmHg. On auscultation
of precordium, there is a grade 3 ejection systolic murmur heard best in the left 4 th
intercostal area, and spleen is palpable 3 cm below subcostal margin. All other
systems are normal. The most likely diagnosis is:
A Infective endocarditis
B Cardiac failure
C Pleural effusion
D Pericarditis
Ans: A
All children of any age with congenital heart disease (except secundum ASD),
including neonates, are at risk of infective endocarditis. The risk is highest when
there is a turbulent jet of blood, as with a VSD, coarctation of the aorta and PDA or if
prosthetic material has been inserted at surgery;
14. The mother of a 6 months old infant has come to you for advice on weaning. The
child has been on exclusive breast feeds until now. The correct advice will be:
15. A 6 years old girl has been brought with history of lethargy and constipation for
the past few months. She is afebrile, a little pale, and answer questions in a low
hoarse voice very slowly. She has a scar in front of the neck where a surgery was
done about 6 months ago to remove a nodule that had been there since birth. The
investigation of choice to arrive at a diagnosis is:
A MRI brain
B Serum urea and creatinine level
C Thyroid function test
D Liver function test
Ans: C
Doc 30
1. A 5-month-old baby boy was brought to klinik kesihatan by his mother with pallor
which she noticed since he was two months old. The baby was exclusively breast
fed from birth. On examination the baby looked pale and had a grade 2/6 murmur in
the left parasternal area. Liver was palpable 3.5 cm below the right costal margin,
and the spleen was just palpable. Blood count showed: Hb 7 g/dL; total white cell
count: 9.0 x 10/μL and platelet count: 170 x 10/μL. Blood film showed anisocytosis,
poikilocytosis, and microcytic and hypochromic red blood cells. No blast cell was
seen.
Name the investigation that should be done to confirm the diagnosis.
a.Haemoglobin electrophoresis
b.Serum Ferritin
c.Serum Iron
d.Total Binding Iron Capacity
Name the most common pathogenic agent that has caused the disease.
a.Adenovirus
b.Respiratory syncytial virus
c.Echovirus
d.Pneumococcus
RSV is the most common cause of lower respiratory tract infection in infants and
children younger than 2 years. Bronchiolitis is the main clinical presentation, followed
by pneumonia.
The incubation period for RSV is typically 2-8 days (mean, 4-6 days). RSV infection
is limited to the respiratory tract. Initial infection in young infants or children
frequently involves the lower respiratory tract and most often manifests as the clinical
entity of bronchiolitis; PE findings include the presence of rhinorrhea, tachypnea,
intercostal, and subcostal retractions
Supportive therapy is the mainstay of care for RSV. Such therapy may include
administration of supplemental oxygen (guided by arterial blood gas values, oxygen
saturation, respiratory rates, and work of breathing, as indicated), fluid replacement,
and mechanical ventilation, as necessary
3.A 5-year-old boy came to the paediatric neurology clinic for consultation for
difficulty in walking. He has to use his hands and arms to "walk" up his own body
while rising from a squatting position.
4.The following statement is true regarding treatment of acute kidney injury (acute
renal failure):
5. A 12-year old boy came to the emergency department with a history of recurrent
swelling of the left knee joint for the last 7 years. Each episode of swelling improved
after he was given Factor VIII infusion. His maternal grandfather has the same
symptom but the swelling often occurred in his right elbow
6. A baby was born with cataract, patent ductus arteriosus and blueberry muffin
lesion on the skin.
Name the most likely organism that caused this clinical picture:
a.Rubella
b.Toxoplasma
c.Cytomegalovirus
d.Herpes virus
Ans: A
blueberry muffin baby presents with distinctive red-purple macules, papules, and
plaques that have historically prompted consideration of congenital infections
recalled by the TORCH acronym: Toxoplasmosis, Other, Rubella, Cytomegalovirus
and HSV
Rubella: Infection before 8 weeks’ gestation causes deafness, congenital heart
disease, and cataracts in over 80% of cases
7.A 5-year old girl came to the paediatric clinic with high fever for 6 days. On
physical examination she had left cervical lymphadenopathy that was non tender,
mobile, measuring 6 cm and non-purulent hyperemic conjunctiva. She also had a red
inflamed tongue, pharynx and tonsils. Her hands were swollen.
12. An 8-year old boy came to klinik kesihatan with the following hepatitis B serology
results: positive HBsAg positive IgM anti HBC and negative anti-HBs.
14. Sivan is a 7-year-old boy who had fever for 5 days and lethargy since this
morning. He had abdominal pain and vomiting yesterday, but no diarrhea. On
examination he is conscious, lethargic, and afebrile. His pulse rate is 138/minute and
respiratory rate is 34/minute. His peripheries are cold and blood pressure is 86/58
mm Hg. He has guarding in right hypochondrium but no other positive findings.
16. A 16-month-old child is brought from a village with a history of passing 10-15
loose stools daily for the last two days. He also had 5-6 vomiting on the first day of
illness. On examination he is afebrile, drowsy, with cold extremities, and has a
capillary refill time of 6 seconds. His mother says she cannot remember when he
passed urine last.
17. A 10-year-old girl has been admitted with history of mild low grade fever
intermittently and breathlessness for the past 3 weeks. She was operated for
complex congenital heart disease at the age of 18 months and again at 7 years. On
examination she is pale and has a temperature of 380 C. Her pulse rate is 126
beats/minute, respiratory rate is 22 breaths/minute and blood pressure is 104/60
mmHg. On auscultation of the precordium there is a grade 3 ejection systolic murmur
heard best in the left 4th intercostal area, and spleen is palpable 3 cm. The rest of
the systems are normal.
19. A 5-month-old infant has high fever, lethargy, seizures and loss of
consciousness. Examination of CSF confirms the diagnosis of meningitis.
20. A 6-year-old boy is brought into A&E by his parents in acute respiratory distress.
He is known to be asthmatic. His mother says that he has needed his inhaler a lot
more than normal over the last 24 h On examination he is tachypneic with a
respiratory rate of 50. He is using his accessory muscles of respiration, and has an
audible wheeze. He is able to talk only in single words. His saturations are 85% and
his heart rate is 160 beats per minute. On arrival in A&E he was started on high flow
oxygen and has already been given a salbutamol nebuliser.
5. A previously healthy 9-month old male infant is admitted with history of low-grade
fever, cough and running nose for 3 days, difficulty in breathing since last evening.
On examination, he is febrile, has a respiratory rate of 58/minute, a heart rate of
142 beats/minute, and warm hand and feet. On examination of the chest, you can
hear rhonchi all over the chest. The first step in his management should be:
a. Send him for a chest X-ray.
b. Give salbutamol nebulisation.
c. Check oxygen saturation with a pulse oximeter.
d. Send blood for culture and sensitivity.
6. A concerned mother brings her 6-month-old baby girl to the clinic. She thinks
her infant is not growing as well as her 2 older children did. She gets very
tired very easily and is not able to finish the feed given to her. In the history
given by the mother, what other symptoms may indicate the child has a
congenital cardiac defect?
a. Pedal oedema
b. Frequent wet diapers
c. Frequent lung infections
d. Abdominal distension
7. A 7-year-old child was referred to the paediatric clinic for a heart murmur from the
school health team. The child was asymptomatic and has normal effort tolerance.
On examination, his vitals were normal, the major pulses were normal, and
cardiovascular examination showed his apex beat at 5th intercostal space at
midclavicular line. The heart sounds were normal and a loud pan-systolic murmur
of grade 4 was heard at the lower sternal edge. What is the most likely diagnosis?
a. Patent ductus arteriosus
b. Ventricular septal defect
c. Atrial septal defect
d. Innocent murmur
9. A 5-month-old girl infant was brought to the clinic by her parents as she felt warm
to touch. They recorded her temperature as 38.6 °C at home. On examination,
she looked well, her temperature was 38.8 °C and all her systems were normal.
Her total white cell count was 18,500/μL, and haemoglobin was 12 g/dL. The best
course of management for her should be:
a. Admit, send urine and blood for culture and start antibiotics.
b. Admit for observations and recording of temperature.
c. Start oral antibiotics as an outpatient for infection.
d. Observe as outpatient with daily follow-up.
10. The following statement is true regarding treatment of acute kidney injury (acute
kidney failure):
a. Daily intravenous fluids ensure good urine output.
b. Diuretic challenge is a useful therapeutic test.
c. Peritoneal dialysis is not an effective treatment.
d. Treatment with IV infusion insulin and glucose reduced hyperuricaemia.
11. A 5-year-old came to the paediatric clinic with high fever for 6 days. On
physical examination, she had left cervical lymphadenopathy that was non
tender, mobile, and measure 4 cm. She also has non-purulent hyperaemic
conjunctiva and red inflamed tongue, pharynx and tonsils. Her hands were
swollen. State the most likely diagnosis of the patient.
a. Scarlet fever
b. Measles
c. Acute viral conjunctivitis
d. Kawasaki disease
12. A 5-year-old previously healthy boy presented with pain and fever for 15 days.
On physical examination, severe pallor, cervical lymphadenopathy and a
palpable spleen were noted. He was treated with antipyretics and blood sample
was sent for full blood count that showed haemoglobin of 5.3 g/dL, leukocytes of
31.07 × 109/L, and platelet count of 121.0 × 109/L. The diagnostic test in this
case should be:
a. bone marrow aspiration.
b. lymph node biopsy.
c. haemoglobin electrophoresis.
d. peripheral blood film.
13. A 5-day-old male infant is brought for check-up because his mother noticed he
looked yellow for the past 2 days. On examination, he was active, feeding well
and passing urine and stools normally. He has jaundice on his face. All systems
are normal on examination. Mother’s blood group is B+ and baby’s blood group
is O+. What is the most likely cause of his jaundice?
a. Blood group incompatibility
b. Neonatal sepsis
c. Hypothyroidism
d. Physiological jaundice
14. A 9-year-old boy presented with a 2-week history of increased bruising and an
episode of epistaxis once yesterday. Last year, he had a tonsillectomy with no
excess bleeding. There is no family history of bleeding disorders. His physical
examination is unremarkable. His laboratory results are as follows (normal
values are given in brackets):
At this time, what is the most appropriate treatment for this patient?
a. Reassurance re-check blood counts after 2 weeks.
b. Bone marrow aspiration and biopsy.
c. Intravenous immune globulin.
d. Short course of prednisone.
15. A previously healthy 5-month-old infant is brought to the emergency with
history of fever for 3 days and recurrent seizures for the past 2 days. The
baby is cared for in a day care nursery. On the second day of fever, the
caretaker called the mother and said the baby has a blank stare for a few
minutes. On bringing the baby home he had several such episodes of blank
stares with stiffening of limbs, each episode lasting for a few minutes.
On examination, he was febrile (38.2 °C), and his SpO2 was 90% in room air.
The heart rate was 88 beats/minute, respiration was shallow at 28/minute. He
was given oxygen inhalation at 2 litres/minute.
16. A 4-year-old girl admitted with history of recurrent low-grade fever for the past 4
months is suspected to have progressive primary tuberculosis. Her father is
being treated for pulmonary tuberculosis for the past 3 months. Investigation to
confirm the diagnosis will be:
a. erythrocyte sedimentation rate
b. X-ray chest, posterior-anterior view
c. gastric lavage for acid fast bacilli
d. Mantoux test
18. A 11-year-old boy was admitted with post-varicella cerebellitis. The neurological
signs to be elicited in him include:
a. intention tremors
b. loss of proprioception
c. high stepping gaits
d. hyperreflexia
19. An infant can babble and has an ulnar grasp. About 2 weeks ago he has started
transferring objects from one hand to the other. He should also be able to:
a. say “mama” and “dada”.
b. sit unsupported.
c. build a tower of 2 blocks.
d. walk with support.
20. A 5-month-old baby boy is brought to Klinik Kesihatan by his mother with
pallor which she has noticed since he was 2 months old. The baby is
exclusively breast fed since birth. On examination, he looks pale and has a
grade 2/6 murmur in the left parasternal area. Liver is palpable 3.5 cm below
the right costal margin, and spleen is just palpable. Blood count shows
haemoglobin 7 g/dL, total white cell 9.0 × 103/μL, and platelet count 170 ×
103/μL. Blood film shows anisocytosis, poikilocytosis, and microcytic red blood
cells. No blasts cell seen.
ANS: A
ANS: D
3.A 7-year-old boy is brought to the hospital following complaints of headache and
vomiting. His blood pressure is 130/90 mmHg. He was treated for skin infections 3
weeks ago. His mother also noted the child to have a decrease in his urine output
over the last 24 hours. Which of the following blood investigations will be most
relevant to confirm the diagnosis?
Select one:
a. Full blood count
b. Renal function tests
c. Anti-streptolysin O titre
d. Complement C3 level
ANS: C
ANS: B
5.A previously healthy 9-month-old male infant is admitted with history of low-grade
fever, cough and running nose for three days, and difficulty in breathing since last
evening. On examination he is afebrile, has a respiratory rate of 58/minute, a heart
rate of 142/minute and warm hands and feet. On auscultation of the chest you can
hear rhonchi all over the chest. The first step in his management should be:
Select one:
a. Give salbutamol nebulization
b. Check oxygen saturation with a pulse oximeter.
c. Send him for a chest X-ray.
d. Send blood for culture & sensitivity
ANS: B
6. A concerned mother brings her six-month-old baby girl to the clinic. She thinks her
infant is not growing as well as her two older children did. She gets tired very easily
and is not able to finish the feed given to her. In the history given by the mother what
other symptoms may indicate the child has a congenital cardiac defect?
Select one:
a. Frequent lung infections
b. Abdominal distension
c. Frequent wet diapers
d. pedal edema
ANS: A
7. A7-year-old child was referred to the Paediatric Clinic for a heart murmur from the
school health team. The child was asymptomatic and had normal effort tolerance. On
examination, his vitals were normal, the major pulses were normal, and
cardiovascular examination showed his apex beat at 5th intercostal space at
midclavicular line. The heart sounds were normal and a loud pansystolic murmur of
grade 4 was heard at the lower sternal edge.
What is the most likely diagnosis?
Select one:
a. Innocent murmur.
b. Patent ductus arteriosus
c. Atrial septal Defect
d. Ventricular Septal defect.
ANS: D
9. A 5-month-old girl infant was brought to the clinic by her parents as she felt warm
to touch; they recorded her temperature as 38.6°C at home. On examination she
looked well, her temperature was 38.8°C and all her systems were normal on
examination.
Her total white cell count was 18,500/μl, and Hb was 12 gm/dL. The best course of
management for her should be:
Select one:
a. Observe as outpatient with daily follow-up
b. Admit for observation and recording of temperature
c. Admit, send urine and blood for culture & start antibiotics
d. Start oral antibiotics as an outpatient for infection
Your answer is correct.
ANS: C
10.The following statement is true regarding treatment of acute kidney injury (acute
renal failure):
Select one:
a. Diuretic challenge is a useful therapeutic test
b. Daily intravenous maintenance fluids ensure good urine output
c. Peritoneal dialysis is not an effective treatment
d. Treatment with I.V. insulin and glucose reduces hyperuricaemia
ANS: A
11.A five-year-old previously healthy boy presented with pain in lower limbs and
fever for 15 days. On physical examination severe pallor, cervical lymphadenopathy
and a palpable spleen were noted. He was treated with antipyretics and blood
sample was sent for full blood count that showed a Hb of 5.3 g/dL, leukocytes of
31.07 x 109/L and a platelet count of 121.0 x 109/L. The diagnostic test in this case
should be:
Select one:
a. Peripheral blood film
b. Lymph node biopsy
c. Haemoglobin electrophoresis
d. Bone marrow aspiration
ANS: D
12.A 5-day old male infant is brought for check-up because his mother noticed he
looked yellow for the past 2 days. On examination he was active, feeding well and
passing urine and stools normally. He had jaundice on his face. All systems are
normal on examination. Mother's blood group is B+ and baby's blood group is O+.
What is the most likely cause of his jaundice?
Select one:
a. Hypothyroidism
b. Physiological jaundice
c. Blood group incompatibility
d. Neonatal sepsis
Your answer is correct.
ANS: B
13.A 9-year-old boy presented with a two-week history of increased bruising and an
episode of epistaxis once yesterday. Last year he had a tonsillectomy with no
excessive bleeding. There is no family history of bleeding disorders. His physical
examination is unremarkable. His laboratory results are as follows (normal values
given in brackets):
At this time, what is the most appropriate treatment for this patient?
Select one:
a. Short course of prednisone
b. Intravenous immune globulin
c. Reassurance, recheck blood counts after 2 weeks
d. Bone marrow aspiration and biopsy
ANS: C
14. A previously healthy 5-month-old infant is brought to the emergency with history
of fever for 3 days and recurrent seizures for the past two days. The baby is cared
for in a day care nursery. On the second day of fever the caretaker called the mother
and said the baby had a blank stare for a few minutes. On bringing the baby home
he had several such episodes of a blank stare with stiffening of limbs, each episode
lasting for a few minutes.
On examination he was febrile (38.2° C), and his SPO2 was 90% in room air. The
heart rate was 88/beats per minute, respiration was shallow and 28/minute. He was
given oxygen inhalation at 2 liters/minute.
15. A 4-year old girl, admitted with history of recurrent low-grade fever for the past 4
months is suspected to have progressive primary tuberculosis. Her father is being
treated for pulmonary tuberculosis for the past 3 months. Investigation to confirm the
diagnosis will be:
Select one:
a. Mantoux Test
b. X-ray chest Posterio-Anterior view
c. Gastric lavage for Acid-Fast-Bacilli
d. Erythrocyte sedimentation rate
ANS: C
17. An 11-year-old boy was admitted with post-varicella cerebellitis. The neurological
signs to be elicited in him include:
Select one:
a. Hyperreflexia
b. Loss of proprioception
c. Intention tremors
d. A high stepping gait
ANS:C
18. An infant can babble and has an ulnar grasp. About 2 weeks ago he has started
transferring objects from one hand to the other. He should also be able to:
Select one:
a. Sit unsupported
b. Build a tower of two blocks
c. say "Mama" and "Dada"
d. Walk with support
ANS: A
19. A 5-month-old baby boy is brought to klinik kesihatan by his mother with pallor
which she has noticed since he was two months old. The baby is exclusively breast
fed since birth. On examination he looks pale and has a grade 2/6 murmur in the left
parasternal area. Liver is palpable 3.5 cm below the right costal margin, and the
spleen is just palpable. Blood count shows: Hb 7 g/dL; total white cell count: 9.0 x
103/μL and platelet count: 170 x 103/μL. Blood film shows anisocytosis,
poikilocytosis, and microcytic and red blood cells. No blast cell
seen.
Name the investigation that should be done to confirm the diagnosis:
Select one:
a. Hemoglobin electrophoresis
b. Serum iron
c. Total iron binding capacity
d. Bone marrow aspiration
ANS: A
Doc 36
1. A 2-month-old is brought to the hospital due to the fever and generalised tonic clonic
seizure for 5 minutes, postictally he was unconscious. On physical examination at the
emergency paediatric department, the infant look lethargy with bulging fontanel.
lumbar puncture was done and the infant was diagnosed as acute bacterial
meningitis. what is the most likely finding on the infant CSF?
a. Clear CSF color
b. Turbid CSF color
c. Xantochromic CSF color
d. Bloody CSF color
2. a mother complains that she is awakened in the night by her 2-year-old son who
developed noisy breathing, marked retractions of the chest wall, and barking cough.
he has upper respiratory tract infection for 2 days. The most likely diagnosis is:
a. Viral Croup
b. Acute bronchiolitis
c. Foreign bodies at the right bronchus
d. Acute epiglottitis
3. A 5-month old Sarah is referred by her general practitioner with pyrexia and flu. Her
parents report that she has been having spiking temperatures for the past 2 days and
is feeding less than usual. She has been vomiting, but there is no history of
diarrhoea. She has been previously fit and well. On examination, she has a
temperature of 38.8’C. She is sleeping but easily arousable. She has a normal cry,
normal movement and a soft fontanel. She has a full, wet nappy. Systemic
examination does not reveal any abnormalities and her vitals are normal. What
further test will be the most appropriate at this time?
a. Chest X-ray AP view
b. Lumbar puncture for CSF studies
c. Nasopharyngeal aspirate for PCR
d. Urine for microscopy and culture
4. A 5-year-old boy is brought to Klinik Kesihatan with fever, jaundice, abdominal pain
and vomiting. He had history of eating seafood with his family 3 weeks ago. His
father has similar symptoms as him. On abdominal examination he has tender
hepatomegaly. Name the blood investigation to confirm the diagnosis.
a. IgM anti HAV
b. HBsAg
c. HCV RNA
d. Anti-HBs
5. An 8-year-old boy came to Klinik Kesihatan with the following hepatitis B serology
results : positive HBSAg; positive anti HBc IgM and negative anti-MBS. Interpret the
serology result.
a. He has chronic Hepatitis B Infection
b. He is immune to Hepatitis B due to natural infection
c. He has acute Hepatitis B infection
d. He is immune to Hepatitis B due to vaccination
8. A 4-year old boy was brought to the emergency unit by his mother with complain of
right. swollen knee. The boy have previously 2 days fever. He suffered recurrent joint
swollen 4 times during this month, involving elbow and ankle joint which resolve after
Factor VIII was given. His Grandfather had the similar symptoms with him, but not his
father. On physical examination the right joint knee was swollen, warm to touch and
have limitation in range of motion. Name the diagnosis of the patient.
a. Hemophilia A
b. Rhematoid arthritis
c. Polyarthritis migrant
d. Acute leukemia
9. A 16-month-old child is brought from a village with history of passing 10-15 loose
stools daily for the last two days. He also had 5-6 vomiting on the first day of illness.
On examination he is afebrile, drowsy, with cold extremities, and has a capillary refill
time of 6 seconds. His mother says she cannot remember when he passed urine
last.His blood investigations are likely to reveal:
a. hypokalemia
b. hyperglycemia
c. Hypophosphatemia
d. metabolic acidosis
10. Amir is a 10-month-old boy who had his BCG and hepatitis B vaccinations at birth.
Since then he has not received any further vaccinations. Which of the following
immunisation schedule would induce him to develop immunity against hepatitis B ?
a. The routine hepatitis B vaccination schedule should be continued from
where it lapsed
b. Amir needs to restart the hepatitis B vaccination schedule all over again
c. Amir needs an extra dose of hepatitis B at 18 months after completing the
schedule
d. Vaccination schedule may be accelerated by giving a dose today and another
a month later
12. A 6-year old boy is brought into A&E by his parents in acute respiratory distress. He
is known to be asthmatic. His mother says that he has needed his inhaler a lot more
than normal over the last 24hrs. On examination, he is tachypnoeic with a respiratory
rate of 50 breaths per minute. He is using his accessory muscles of respiration, and
has an audible wheeze. He is able to talk only in single words. His oxygen saturation
is 85% and his heart rate is 160 beats per minute. On arrival in A&E, he is started on
high flow oxygen and given nebulised salbutamol once. The next step in
management should be:
a. Give a second salbutamol nebulisation
b. Secure IV access and give a fluid bolus
c. Check his arterial blood gas levels
d. Give inhaled steroids
13. A 10-year-old girl has been admitted with a history of low grade intermittent fever and
breathlessness for the past 3 weeks. She was operated for complex congenital heart
disease at the age of 18 months and again at 7 years. On examination she is pale
and has a temperature of 38° C. Her pulse rate is 126 beats/minute, respiratory rate
is 22 breaths/minute and blood pressure is 104/60 mmHg. On auscultation of the
precordium there is a grade three ejection systolic murmur heard best in the left 4th
intercostal area, and spleen is palpable 3 cm. All other systems are normal. The most
likely diagnosis is:
a. Infective endocarditis
b. Cardiac failure
c. Pleural effusion
d. Pericarditis
14. The following statement is true regarding treatment of acute kidney injury (acute
renal failure):
a. Daily intravenous maintenance volume to maintain good urine output
b. Fluid challenge is a useful therapeutic test
c. Treatment with insulin glucose is a strategy to reduce hyperuricemia
d. Peritoneal dialysis is not an effective treatment
16. A 1-year-old boy is brought to Klinik Kesihatan for 3 days for fever, vomiting 3 times
daily and profuse non bloody and non mucousy watery diarrhea. The boy looks very
thirsty and has reduced urine output. On physical examination his body weight is 9.2
kg (it was 10 Kg 5 days ago). He is conscious, irritable and has dry lips and tongue.
His skin turgor is less than 3 seconds. The hydration status of the child is
a. Moderate dehydration
b. No dehydration
c. Mild dehydration
d. Severe dehydration
17. A 5-month-old baby boy was brought to the Klinik Kesihatan by his mother with pallor
which she noticed since he was two months old. The baby was exclusively breast fed
since birth. On examination the baby looked pale and had a grade 2/6 murmur in the
left parasternal area. The Liver was palpable 3.5 cm below the right costal margin,
and spleen was just palpable. The full blood count showed low hemoglobin but
normal total white cell count and platelet count. His blood film showed anisocytosis,
poikilocytosis, and microcytic and hypochromic red blood cells. No blast cell was
seen. The investigation that should be done to confirm the diagnosis is
a. haemoglobin electrophoresis
b. serum ferritin
c. serum iron
d. total iron binding capacity
18. A 3-year old girl was brought to the hospital with intermittent fever of three days
duration. Urine dipstick investigation showed negative leucocyte esterase but positive
nitrite. State the most appropriate management in this child.
a. Give antipyretic, assure the parents and send the patient home.
b. Start antibiotic if the urine test was carried out on a fresh urine sample
c. Admit the patient and wait for blood culture and sensitivity results
d. Advise to take adequate oral fluid and come again in the next five days
19. Sivan is a 7-year-old boy who had fever for 5 days and lethargy since this morning.
He had abdominal pain and vomiting yesterday, but no diarrhea. On examination he
is conscious, lethargic and afebrile. His pulse rate is 138/minute and respiratory rate
is 34/minute. His peripheries are cold and blood pressure is 86/58 mm Hg. He has
guarding in right hypochondrium. An additional clinical finding characteristic of
Dengue fever is:
a. Petechial rash
b. Splenomegaly
c. Coarse crackles in lungs
d. Arthralgia
Doc 37
1. As a part of the Expanded Immunization Program in Malaysia, all babies would receive
Hepatitis B vaccine at birth.
Which of the statements below is true regarding the vaccine?
13. A 2 year-old child is on regular follow up in the cardiac clinic for newly diagnosed Tetralogy of
Fallot. He is on the waiting list to have surgical correction to be done by the end of the year.
Meanwhile he has been noted to have intermittent increase of his cyanosis recurring three to four
times a week. These “hyper cyanotic spells” can be best explained by
14. A 12-year old boy has been having intermittent fever almost every third day in the last two weeks,
but has not sought any medical treatment. He comes from a small village which is a dengue endemic
area and has not received any immunization at all.
Examination does not reveal any abnormalities except for the fever and dull Traube space on
percussion. The most reasonable provisional diagnosis is
19) A six- year-old girl complains of abdominal pain frequently early in the mornings. This has been
going on for about a month now. There is no diarrhea or vomiting and she is active as usual. She is
afebrile and all her systems are normal on examination. You will investigate for the chronic abdominal
pain if:
An 8- year- old girl with Down syndrome presented to the outpatient clinic. Clinically on examination
she had central cyanosis and digital clubbing.
There was no respiratory distress and the apex beat was normal. The heart sounds were normal first
heart and single second sound. There was ejection systolic murmur grade 3 loudest at the pulmonary
area. Other cardiovascular examination was unremarkable. What is the most likely diagnosis?
A 9-year-old girl was noted by her mother to have abnormal eye movements about three weeks after
having chicken pox. A diagnosis of post chickenpox encephalitis affecting mainly her cerebellum is
best suggested by the following EXCEPT:
a. hypertonia of her limbs
b. abnormal deep tendon reflexes
c. fine tremors of the hands
d. past pointing on finger nose test
a. Transverse myelitis
b. Guillain-barre syndrome
c. Poliomyelitis
d. Myasthenia gravis
10. The following are the complications of a large ventricular septal defect
EXCEPT:
19. A 4-year-old boy is admitted with acute gastroenteritis with severe dehydration and shock. He has
been infused with adequate intravenous fluids in the past five hours, and his vital signs are now
stable. However, he has not passed urine since admission. Since he was passing frequent watery
stools, his mother does not know when he passed urine last. The next step in management should be
A 13-month-old baby is brought to the child health clinic for immunization. She missed first MMR.
(9,12) The mother has brought the child because of the measles outbreak in her town and wants her
child to be immunized. According to the mother the baby has egg allergy. She is also on her second
month steroid treatment for nephrotic syndrome.
What is your advice?
A 5-year-old child was suspected to have a food allergy. The following feature(s) suggest that the
pathology is IgE mediated allergy
25. An 11-month-old boy was noted to be pale on his well-child checkup. He is still exclusively
breastfeeding. His hemoglobin 78 g/L (N: 110 - 140 g/ L); hematocrit 22.9% (N: 31 - 41 %); mean
corpuscular volume 64 fL. (N: 7 - 80 fL) and reticulocyte count 2 %. Peripheral blood smear showed
microcytosis and hypochromia. Which of the following is the most appropriate management?
14. A 6-year-old boy was brought to the clinic for pallor, fever, and gum bleeding for 1 week. On
physical examination, his temperature was 38°C; his liver and spleen were enlarged.
Hb was 80 g/L; WBC 20 x 109/L; platelet count 90 x 109/L.
The investigation that would confirm the diagnosis is:
a. ALL needs prolonged maintenance treatment (two to two and one half-years)
b. Chronic myeloid leukemia does not occur in children.
c. Bone marrow studies are mandatory for assessing remission.
d. Acute lymphoblastic leukemia (ALL) is the commonest type.
A 5-year-old boy presents with persistent dull abdominal pain of about 3 weeks duration. There was no history of
trauma or diarrhoea and vomiting. His appetite remains well. On examination there is no pallor or jaundice.
Hepatomegaly of 4 cm below the right subcostal margin is noted. The following features strongly support the
diagnosis of hepatoblastoma EXCEPT:
A. Elevated serum alpha fetoprotein (primary tumour marker for hepatoblastoma)
B. Lung nodules on the CT scan of the chest (liver mets)
C. Discrete nodular mass in the liver
D. Positive Hepatitis B surface antigen
10. A 6-month-old exclusively breast-fed female is admitted to the hospital from her primary care physician’s
office. She has had 8-10 episodes of diarrhoea since this morning. Her weight in the office is 8.0 kg which has
decreased from her weight in the office of 8.6 kg just three days ago during a well child check. Her temperature is
37.8°C; she is lethargic, pale and her eyes are sunken. Her anterior fontanel is soft. All systems are normal on
examination. Bowel sounds are hyperactive. The most likely causes of her diarrhoea:
A. Escherichia coli
B. Salmonella para typhi
C. Rotavirus
D. Milk protein allergy
A 10-year-old girl has been referred to paediatric clinic for rapid weight gain in the past ten months from 23 kg to
36 kg without significant gain in height. The girl has moon facies, acne on face, striae on the abdomen and
hypertension. What is the best laboratory investigation to diagnose the patient?
A. Serum cortisol
B. Serum cholesterol
C. Serum sodium
D. HbA1C
Ans: Cushing's syndrome
15. A previously healthy 5-month-old infant is brought to the emergency with history of fever for 3
days and recurrent seizures for the past two days. The baby is cared for in a day care nursery. On the
second day of fever the caretaker called the mother and said the baby had a blank stare for a few
minutes. On bringing the baby home he had several such episodes of a blank stare with stiffening of
limbs, each episode lasting for a few minutes. On examination he was febrile (38.20 C), and his SPO,
was 90% in room air. The heart rate was 88/beats per minute, respiration was shallow and 28/minute.
He was given oxygen inhalation at 2 liters/minute.
The immediate next step should be:
a. An electro-encephalography
b. ultrasonography of the brain
c. CT scan of brain
d. A cerebrospinal fluid analysis